CEN Study Questions

¡Supera tus tareas y exámenes ahora con Quizwiz!

A 30-year-old patient who has had a cold for approximately 10 to 14 days now has a paroxysmal cough with loud end-cough whooping noises, a low-grade fever, and anorexia. The nurse should prepare to? A) assist with endotracheal intubation. B) immunize the patient with pertussis vaccine. C) admit the patient to the general medical floor. D) administer IV fluids and antibiotics.

D For a patient with pertussis and decreased oral intake, the nurse should establish IV access for administration of crystalloid solution and administer antibiotics because it is a bacterial infection.

A patient presents with purulent nasal discharge and a frontal area headache that worsens when bending over. Which of the following nursing interventions is MOST effective? A) Place the patient in a supine position. B) Perform nasal lavage. C) Provide a darkened environment. D) Apply heat to the face.

D Frontal pain exacerbated when bending over, headache and purulent nasal discharge indicate sinusitis. Treatment includes nasal/systemic decongestants, nasal steroids, keeping the head of the bed elevated to promote sinus drainage and applying heat to the face to relieve pressure.

What is the PRIORITY treatment for a patient with signs and symptoms of heat exhaustion? A) Administer oxygen. B) Establish IV access. C) Obtain blood samples. D) Initiate cooling measures.

D Greater loss of motor function in the upper rather than the lower extremities are associated with central-cord syndrome.

In which of the following cases has the nurse breached patient confidentiality? A) notifying the health department about measles B) reporting sudden unexpected deaths to the coroner C) notifying the police about a possible homicide D) reporting HIV status to the patient's employer

D HIV status of a patient is confidential patient information and may not be provided to the patient's employer. The other three options are mandated reportable occurrences the emergency department nurse may encounter.

The nurse is caring for a patient with a hyphema. The nurse knows the BEST way to position this patient is? A) supine B) Trendelenburg C) prone D) head of bed elevated

D Having the head of bed elevated 30-45 degrees will decrease intraocular pressure. Positioning the patient supine, prone or in Trendelenburg can increase intraocular pressure.

Which clotting disorder is caused by the absence or deficiency of factor IX? A) hemophilia C B) Von Willebrand Disease C) hypercoagulation D) hemophilia B

D Hemophilia B is caused by the absence or functional deficiency of factor IX.

A patient who has missed the last three dialysis appointments is found to have peaked T-waves and runs of intermittent ventricular tachycardia on the ECG. Which laboratory finding would be MOST consistent with this patient's presentation? A) serum sodium level of 135 mEq/L B) serum chloride level of 98 mEq/L C) serum calcium level of 7.9 mg/dL D) serum potassium level of 7.5 mEq/L

D Hyperkalemia, often caused by renal failure, can cause cardiac irritability.

The nurse is caring for a patient who is complaining of eye pain and burning after having a chemical splashed in the face. The nurse's IMMEDIATE action should be to? A) apply an eye patch. B) insert an intravenous line. C) assess visual acuity. D)irrigate the eyes.

D Immediate irrigation should begin as soon after the injury as possible. Assessing visual acuity, applying an eye patch and inserting an intravenous line are not appropriate immediate actions.

A confused elderly woman has a blood glucose level of 800 mg/dL. Serum ketones are negative and arterial blood gases show a pH of 7.25. The nurse should anticipate an order to administer which of the following FIRST? A) sodium bicarbonate B) high-dose insulin C) potassium chloride D) large volume of IV fluid

D Increased glucose, negative ketones, and acidosis are signs and symptoms of hyperosmolar hyperglycemic nonketotic coma. The treatment goals are rehydration and correcting electrolyte imbalances prior to potassium replacement and insulin.

The nurse is caring for a patient who has a high fever, central nervous system dysfunction and tachycardia. The nurse should recognize that the patient may be experiencing? A) Beck's triad. B) hyperosmolar hyperglycemic nonketotic syndrome (HHNS). C) myxedema crisis. D) thyroid storm.

D The classic triad of thyroid storm is high fever, central nervous system dysfunction or cerebral encephalopathy and exaggerated tachycardia out of proportion to fever.

Which of the following study types has the highest level of evidence appraisal? A) Cohort Studies B) Randomized Controlled Trials C) Case Studies D) Systematic Reviews

D The highest level of evidence is Meta-Analyses, followed by Systematic Reviews. Randomized Control Trials, Case Studies, and Cohort Studies are lower levels of evidence because they are considered 'Unfiltered Information'.

An adolescent patient presents with symptoms of depression and suicidal thoughts. The highest priority for the patient is to? A) notify the guardian of the patient for treatment consent. B) initiate medical clearance including blood and urine collection. C) complete intake for pending transfer to psychiatric facility. D) initiate measures for suicide prevention and safety.

D The highest priority for patients with mental health concerns is safety for them and others during their care in the emergency department. Suicidality must be taken seriously and measures initiated to promote safety from self harm. Many states allow minors to initiate care in the Emergency Department without requiring consent from their caregivers (including mental health, reproductive health, etc) so guardian notification is not the highest priority. The patient may be transferred to an inpatient psychiatric facility which would require a completed history, physical and medical clearance, however this is not a higher priority than patient safety and suicide prevention.

The INITIAL drug of choice in the treatment of anaphylaxis is? A) prednisone (Sterapred). B) famotidine (Pepcid). C) diphenhydramine (Benadryl). D) epinephrine (Adrenalin).

D The mechanism of anaphylaxis includes vasodilation that is initially treated with epinephrine. Other medications with other mechanisms of action are used, but do not take priority.

A 21-year-old patient is suspected of having a left-sided spontaneous pneumothorax. The physical finding that MOST strongly supports this diagnosis is A) increased breath sounds on the left. B) an absence of tactile fremitus on the right. C) dullness on percussion of the right. D) hyperresonance on the left.

D With a pneumothorax, air escapes into the pleural space. Auscultating over this area of trapped air will result in diminished or absent breath sounds. When percussing, a hyperresonance will be noted.

An 82-year-old patient sustained a fractured hip in a recent fall. In evaluating the patient's condition, the nurse should be especially concerned about the? A) degree that the affected leg is rotated outward. B) time the fall occurred. C) cause of the fall. D) patient's history of drug allergies.

C Elderly patients are at risk for falls. The etiology of the fall is investigated to rule out a cardiac, vascular, or neurologic origin. The other options are components of the nursing history and assessment. The most important assessment at this time is to determine the etiology of the fall.

A 3-year-old child who has acute respiratory distress is admitted to the emergency department. The child is sitting forward and drooling, and the neck is extended. These findings support the diagnosis of? A) aspiration of a foreign body. B) croup. C) epiglottitis. D) bronchiolitis.

C Epiglottitis is a condition characterized by edema of the epiglottis, which puts the patient at risk for airway obstruction. The patient will find a position that maximizes air exchange. Drooling indicates that edema is interfering with swallowing.

A 35-week pregnant patient presents with right upper quadrant abdominal pain and a low-grade fever for 24 hours. These symptoms suggest? A) appendicitis. B) pancreatitis. C) cholelithiasis. D) nephrolithiasis.

A Appendicitis in pregnancy can have RUQ pain due to the uterus displacing the appendix. Pancreatitis symptoms include fever and epigastric pain, nephrolithiasis symptoms include flank pain and cholelithiasis symptoms include RUQ pain but not fever.

The cardinal signs of facial fractures are pain, swelling, and? A) asymmetry. B) bleeding. C) salivation. D) fractured teeth.

A Asymmetry is the first facial feature to be assessed after the Primary assessment and necessary lifesaving interventions have been completed. Bleeding can occur from injuries even when a fracture is not present. Salivation occurs naturally, and inability to handle secretions alone are not a sign of a facial fracture. Fractured teeth can occur without facial fractures.

Which of the following drugs is commonly used to treat symptomatic bradycardia? A) atropine sulfate B) metoprolol (Lopressor) C) calcium chloride D) diltiazem (Cardizem)

A Atropine sulfate is the drug of choice for the treatment of symptomatic bradycardia. Other therapies for bradycardia include pacing, sympathomimetic agents, and oxygenation (particularly in children). Metoprolol (a beta-blocker) and diltiazem (a calcium channel blocker) are both used to reduce heart rate and cardiac work.

Immediate intervention for a patient who has a large dermal avulsion injury of the arm should focus on? A) controlling hemorrhage. B) preventing infection. C) minimizing the loss of mobility. D) minimizing scarring.

A Avulsions are characterized by full-thickness tissue loss that prevents wound approximation. Hemostasis is the immediate problem. Immediate intervention should focus on controlling bleeding. The distractors are incorrect because controlling hemorrhage needs to occur prior to the ongoing preventive care associated with minimizing scarring, loss of mobility, and preventing infection.

The nurse is assessing a patient who was involved in a motor vehicle accident. The patient has muffled heart sounds, jugular vein distension and dyspnea. Vital signs are as follows:BP 67/49 mm HgHR 120 beats/minRR 28 breaths/minO2 sat 90% on room airThe nurse recognizes that the patient may be experiencing? A) cardiac tamponade. B) ruptured diaphragm. C) laryngeal injury. D) pneumothorax.

A Beck's Triad (hypotension, muffled heart sounds and JVD) is characteristic of cardiac tamponade which can occur after a trauma.

A patient who has a long-leg cast has pain out of proportion to the injury and decreased sensation in the affected leg. Urinalysis reveals myoglobinemia. These findings MOST likely indicate? A) compartment syndrome. B) nerve laceration. C) deep vein thrombosis. D) fat emboli.

A Characteristic signs of compartment syndrome are pain out of proportion of the extent of the injury and decreased sensation. Muscle damage due to compression/damage of the muscle releases myoglobin, which will be evident in the urinalysis.

What life-threatening orthopedic injury is caused by internal soft tissue swelling compromising vascular and neurologic structures? A) compartment syndrome B) osteomyelitis C) fat embolism D) hematoma

A Compartment syndrome occurs when compartmental pressures increase from an internal or an external force. Increased internal pressures result from bleeding into the compartment or internal soft tissue swelling. External forces include rigid casts, splints, air splints and PASGs. As intercompartmental pressures rise, the vascular and neurologic structures become compressed and compromised.

The nurse is providing discharge education to a patient diagnosed with conjunctivitis. The nurse knows that the education is successful when the patient states? A) "I will be sure to wash my hands often." B) "No further follow up is needed." C) "It is ok to wear my contacts since I am already being treated." D) "I will continue to use my eye makeup."

A Conjunctivitis is highly contagious, and patients should follow good infection control measures such as frequent hand washing. Touching the eye, including contact insertion, should be avoided and eye makeup should be discarded due to contamination. The patient should follow up with their health care provider or ophthalmologist in 2 to 3 days.

A patient who receives dialysis treatments is weak and has muscle cramps. Electrocardiogram (ECG) readings show first-degree heart block with peaked T waves. The patient has a heart rate of 70 beats/min and a serum potassium level of 8.0 mEq/L. The nurse should anticipate administering? A) calcium gluconate 1 g, slow IV push. B) lidocaine (Xylocaine) 100 mg, slow IV push. C) atropine 0.5 mg, IV push. D)magnesium sulfate 2 g, IV over 1 hour.

A Dialysis patients are at risk for fluid and electrolyte disorders. Signs and symptoms of hyperkalemia include peaked T waves and first-degree AV block. Treatment includes 10% calcium gluconate to counter the toxic effects of hyperkalemia on the heart.

The nurse is caring for a patient who was found in a garage with a running car. The patient is reporting nausea, a headache and is lethargic. The nurse should recognize that the patient will benefit MOST from administration of? A) dextrose. B) high-flow oxygen. C) intravenous fluids. D) an antiemetic.

B The patient should be treated for carbon monoxide poisoning. 100% high-flow oxygen decreases the half-life of carboxyhemoglobin, which forms in the blood because of the carbon monoxide. The administration of intravenous fluids, dextrose and an antiemetic will have no effect on the level of carboxyhemoglobin.

Which of the following is the MOST common finding in pulmonary embolism? A) crackles (rales) B) dyspnea C) fever D) hemoptysis

B With pulmonary embolus, common signs and symptoms include shortness of breath, tachypnea, tachycardia, and sudden-onset pleuritic chest pain that increases with respirations. Dyspnea is the most common sign of pulmonary embolism. Crackles and hemoptysis are likely to be associated with pulmonary edema. Fever is a finding in patients with pulmonary embolism, but it is not common.

Which situation would MOST likely result in moral distress for an Emergency Department (ED) Nurse? The ED nurse: A) is asked to float to the Intensive Care Unit to assist with staffing needs B) is unable to provide quality care to their patients due to shortages in C) staffing and overcrowding. C) witnesses multiple events where patients have severe traumatic injuries. D) receives the same compensation for overtime shifts as they do for regular shifts.

B Studies have shown that moral distress is frequently caused by the inability of ED Nurses to provide quality care to their patients due to circumstances outside their control, such as staffing or hospital census.

he nurse knows that education following a positive Mantoux Test has been effective when the patient states? A) "I have HIV." B ) "I will need to start antibiotics immediately." C) "I have been infected with tuberculosis at some point." D) "I am contagious and should quarantine."

C A Mantoux Test is a screening test for tuberculosis. A positive result indicates that the patient has been infected with tuberculosis at some point in time, but more tests will be needed to determine if the patient has latent TB or active disease. Latent TB is not contagious, so a positive Mantoux Test does not necessarily mean the patient is contagious. Antibiotics are not indicated based solely on a positive Mantoux screening test.

A patient who has an infection returns to the emergency department. After determining that the patient has not been taking the prescribed antibiotics, the nurse reviews the instructions with the patient. Which of the following responses by the nurse is MOST appropriate? A) If you don't take the antibiotics, you will need to return for injections. B) Here is a list of times you will take the antibiotics. C) Tell me how you will take the antibiotics. D) Do you have any questions about how to take the antibiotics?

C

The nurse is assessing a patient who complains of a sudden headache, nausea and vomiting that started while they were exercising. The nurse knows that the PRIORITY intervention is? A) administration of pain medication. B) a lumbar puncture. C) a CT scan of head. D) administration of antiemetics.

C A CT scan of the head is the priority intervention to quickly rule out a subarachnoid hemorrhage. The patient may require pain and antiemetic medication but they are not the initial priority. If indicated, a lumbar puncture would be completed after the CT scan.

A man who weighs 165 lb (75 kg) arrives at the emergency department immediately after sustaining a 40% partial-thickness burn. Using the formula 4 mL/kg multiplied by the percent of body surface area burned, IV fluids should be administered at what rate during the first 8 hours? A) 750 mL/h B) 1000 mL/h C) 500 mL/h D) 375 mL/h

A Use the Parkland formula: 4 mL/kg/total body surface area of the burn. The first half of the calculated amount of IV fluid should be given during the first 8 hours and the remainder should be given over the next 16 hours. 4 mL x 75 kg x 40% = 12,000/2 = 6,000/8 = 750 mL/h

The nurse is caring for a patient who had a traumatic amputation of the foot. The BEST way to care for the amputated part is to? A) wrap the part in sterile gauze moistened with normal saline. B) place the part in a container with sterile water. C) wrap the part in dry sterile gauze. D) place the part directly in chilled normal saline.

A When caring for an amputated body part the part should be wrapped in sterile gauze and moistened with normal saline or lactated ringers. An amputated body part should not be soaked as it causes a fluid shift and further tissue damage.

A patient who has chest pain and palpitations is given a dose of adenosine (Adenocard) by rapid IV push. The optimal outcome is? A) the supraventricular tachycardia resolves B) a transient decrease in heart rate. C) the chest pain decreases. D) the systolic blood pressure increases.

A A decrease in chest pain and an improvement in systolic blood pressure are benefits that can be derived from adenosine therapy in a patient with supraventricular tachycardia (SVT). However, resolution of the tachydysrhythmia is the desired endpoint and indicates drug effectiveness. A transient decrease in heart rate following adenosine administration, even frank asystole, indicates the drug has reached the heart, but this is not the endpoint of therapy. Adenosine (Adenocard) should be given until conversion occurs or maximum dose is given.

What is the most appropriate intervention for a patient in ventricular fibrillation? A) defibrillation B) transvenous pacemaker C) synchronized cardioversion D) external pacemaker

A In ventricular fibrillation the patient has no organized cardiac activity and no pulse. The early application of an unsynchronized DC countershock is the appropriate response. Pacemakers, external or transvenous, can pace a heart but will not terminate fibrillation. Synchronized countershocks are appropriate for a patient with a discernible R wave (e.g., a patient in supraventricular tachycardia) or a patient in ventricular tachycardia with a pulse.

The emergency nurse has been a victim of workplace violence from an intoxicated patient. In order to pursue legal action, the nurse knows? A) they must personally file criminal charges. B) criminal charges cannot be filed against a patient. C) hospital security will file criminal charges. D) hospital leadership will file criminal charges.

A One person intentionally assaulting another person is considered a crime, whether it be in the hospital setting or not. The emergency nurse should personally report the incident to law enforcement, as a manager, leader, or security officer cannot do it for them.

When caring for a patient with a diagnosis of disseminated intravascular coagulation, the nurse understands? A) medications should be administered intramuscularly B) all invasive lines must be secured C) suctioning should be avoided D) oral care should be restricted.

B Invasive lines should be secured to prevent dislodgement and bleeding. Maintaining a patient airway is important, but suctioning should only be done to keep the airway patent. Medications should be given intravenously to prevent multiple puncture sites. Oral care should be done with a soft bristled brush, not avoided.

The nurse caring for a patient with tuberculosis knows education has been effective when the patient states? A) "This condition is not treatable with medications." B) "I will need to take my tuberculosis medications daily until completed." C) "I will need to take my tuberculosis medications as needed for breakthrough symptoms." D) "I will have to be on antibiotics for the rest of my life."

B It is important that medications for tuberculosis be taken daily until the regimen is complete. This regimen is often as long as 6 months.

The nurse is caring for a patient who has a serum potassium level of 2.2 mEq/L. The nurse would question orders for? A) potassium. B) furosemide (Lasix). C) aldactone (Spironolactone). D) magnesium sulfate.

B Lasix should be given cautiously in hypokalemic patients as Lasix is not potassium-sparing and can cause increased hypokalemia. Aldactone (Spironolactone) is a potassium sparing diuretic. Hypokalemia is associated with hypomagnesemia, and the severity of the hypokalemia correlates with a similar degree of hypomagnesemia. Magnesium replacement should usually accompany potassium repletion.

An elderly woman presents with vague complaints of abdominal pain. The patient states that the people she lives with are stealing from her. The nurse should? A) arrange for protective custody of the patient. B) assess the patient for signs of neglect and maltreatment. C) call the family to discuss the situation. D) obtain consent for evaluation by social services.

B Maltreatment of the elderly cannot be assessed quickly or easily from a cluster of signs and vague presenting symptoms. Keen awareness when performing the physical examination is essential to identify elder abuse.

Which herbal supplement, when combined with a prescribed selective serotonin reuptake inhibitor (SSRI) antidepressant, is known to increase the levels of serotonin in the blood stream and can lead to serotonin syndrome? A) Saw Palmetto B) St. John's Wort C) Evening Primrose D) Black Cohosh

B St. John's Wort, when combined with a prescribed selective serotonin reputable inhibitor (SSRI) antidepressant, can lead to accumulation of high levels of serotonin in the blood stream. Saw palmetto, black cohosh, and evening primrose are not known to increase serotonin levels.

The nurse is caring for a patient who sustained a stingray puncture to their forearm while snorkeling. The ED nurse anticipates? submersion of the arm in cold water. submersion of the arm in hot water. constrictive dressing application. complete wound closure with sutures.

B Stingray venom is heat-susceptible and will be deactivated by submersion in hot water for 60-90 minutes. Cold water and constrictive dressings will not aid to deactivate the stingray venom. Open wound or loose closure with packing, following foreign body removal and copious irrigation, is the recommended treatment due to infection and drainage from the venomous wound.

A trauma patient arrives at the emergency department conscious and with stable vital signs. On initial assessment, which of the following findings is most concerning? A) weak pedal pulses B) subcutaneous crepitus of the left upper chest C) distention of the bladder D) absent bowel sounds

B Subcutaneous crepitus of the left upper chest is suspicious for a pneumothorax. A sequential initial assessment starts with airway followed by breathing. Identification of symptoms of a pneumothorax is a component of the breathing assessment. An untreated pneumothorax will affect the patient's ability to breathe and properly oxygenate, as well as put the patient at risk for total circulatory collapse from a tension pneumothorax. Pedal pulses, bowel sounds, and bladder distention are not initial assessment findings; they would be included in the head-to-toe assessment later in the trauma evaluation. The presence of a pedal pulse, even a weak one, indicates blood flow to the extremity. Therefore, the weak pulse would not be the most concerning finding. Absent bowel sounds may be a sign of profound shock or visceral injury. Either of these may be identified during the circulation phase of a trauma assessment which follows the breathing assessment. Distention of the bladder would not be identified until the head-to-toe assessment, which may indicate a genitourinary injury. This would be addressed after stabilization of the ABCs.

A 10-month-old infant who has a sudden onset of wheezing, cough, and stridor should immediately be assessed for? A) presence of abdominal breathing. B) unequal breath sounds. C) a history of bronchodilator use. D) axillary subcutaneous emphysema on palpation.

B Sudden onset of respiratory distress frequently indicates aspiration of a foreign body, so assessment should include bilateral breath sounds to determine if there is obstruction.

A patient has just been prescribed the "morning after" pill, and the nurse has administered the first dose. Which of the following statements indicates the patient understands discharge teaching? I will take the other two tablets in 24 hours. The medication may make me feel nauseated My menstrual period will occur in 3 to 4 days. Using condoms will prevent me from getting pregnant.

B The "morning after" pill is an estrogen and progesterone birth-control pill that causes nausea and vomiting. It is common for the treating healthcare provider to also prescribe an antiemetic. This treatment will not stimulate a menstrual cycle. Using condoms after the fact will not prevent a pregnancy. The usual course of dosing is 2 pills now and 2 more in 12 hours.

The nurse is caring for a patient who is exhibiting acutely violent behavior. The nurse should? A) offer the patient oral fluids or food to establish rapport. B) ensure the room is safe and request assistance. C) attempt to de-escalate the situation while awaiting backup. D) lock the patient in the room until backup arrives.

B The first priority is safety for the patient and for the nurse. The nurse should never approach a violent patient alone, even in an attempt to de-escalate. When a patient is experiencing a psychiatric emergency that is manifesting in violent behavior, any extra objects should be avoided as they can be used as a weapon--food, cups, utensils, etc. A patient should never be locked or barricaded in the room.

The nurse is caring for a patient who suddenly stops breathing and is in ventricular fibrillation. The nurse should FIRST? A) administer 1 mg of intravenous epinephrine. B) start chest compressions. C) perform synchronized cardioversion at 200 joules. D) provide oxygen and ventilation via a bag-mask device.

B The first step the nurse should take is to start high-quality chest compressions to maintain perfusion to the heart and brain. Oxygenation and ventilation can happen after the first 30 compressions. Epinephrine is also indicated for a ventricular fibrillation, but should not be prioritized over chest compressions. Synchronized cardioversion is not indicated for ventricular fibrillation and defibrillation would be the correct indication.

A patient reports weakness, fatigue, nausea and vomiting. On assessment, the nurse notes hyperpigmentation of the knuckles and elbows. The nurse should suspect? A) hyperthyroidism. B) Addisonian crisis. C) diabetes insipidus. D) diabetic ketoacidosis.

B The following patient symptoms of weakness, fatigue, nausea and vomiting cause Addisonian crisis.

A patient presents with a large piece of metal penetrating from the eye. On assessment, the nurse notes asymmetry of the globe. Which intervention should the nurse prioritize? A) Palpate around the affected eye. B) Secure the foreign body. C) Administer topical ophthalmologic medication. D) Place a patch over the affected eye.

B The foreign body should be secured first then both eyes lightly patched to prevent further injury. Both eyes are patched to minimize consensual movement. Any pressure around the globe and topical medications should be avoided when a globe rupture is suspected.

A patient who has been in a motorcycle crash is awake, alert, and oriented, but complains of severe neck pain. Shortly after arrival, the patient experiences respiratory arrest. Appropriate INITIAL management of the airway should consist of? A) administration of oxygen by a nasal cannula. B) jaw-thrust maneuver and ventilation with a bag-mask device C) endotracheal intubation. D) head-tilt maneuver and placement of an oropharyngeal airway.

B The jaw-thrust maneuver should be used as the basic airway management procedure in any patient with a suspected cervical spine injury and the head should not be tilted. Endotracheal intubation will be done, but it is not the first action to be taken.

Manual uterine massage is recommended for early postpartum hemorrhage because massage will? A) stimulate uterine contractions. B) improve bladder tone. C) enhance uterine atony. D) expel retained products of conception.

A The postpartum uterus may be atonic. Manual stimulation of the uterus will result in the muscles and vessels of the uterus contracting. This will reduce intrauterine bleeding.

An adolescent patient presents to the emergency room with reports that they were hit in the eye with a baseball. No visual loss is noted when a Snellen visual acuity is performed, but while waiting to be assessed by a provider, the patient reports "aching" pain in their eye and sudden onset of blurry vision. What statement made would cause the greatest concern in the emergency nurse? A) Reports everything looks red B) Complaints of increased lacrimation C) Increased swelling D) Decreased ability to look up

A "Seeing red" is an indication that blood is present in the anterior chamber. Activities such as coughing, sneezing, or leaning forward can cause the bleeding to continue or increase. This causes the anterior chamber to fill with blood and can lead to increased intraocular pressure due to blood clotting. Increased swelling and lacrimation are anticipated after an eye injury. Decreased ability to look upward may be a sign on entrapment, however without vision loss, this would take less priority than the chamber of the eye filling with blood.

After an incident involving a young child with a negative outcome, the team is scheduled for a debriefing. The purpose of a formal debriefing is to? A) support staff by allowing time for reflection and communication B) provide a time for the rest of the department to listen to the chain of events of the incident. C) allow staff to explain all the rationales to the hospital's risk management team D) provide staff with an opportunity to explain their actions and rationales to the patient's family.

A A debriefing is a formal response to a critical incident that may have had a profound effect on the staff involved. It is typically held within a few days of the incident and its purpose is to allow a time of open reflection and communication about the incident. It allows the staff to decompress any emotions or stress they may have in regards to the critical incident. The purpose of the debriefing is to support the emotional and psychosocial health of the staff, and not a time for the staff to feel the need to defend their actions to the patient's family or the hospital's risk management team. It is also typically reserved for staff that were directly involved or affected by the critical incident.

Testicular torsion is usually associated with scrotal pain that has a? A) sudden onset associated with nausea. B) sudden onset that resolves after urination. C) gradual onset and an associated fever. D) gradual onset that radiates into the lower abdomen.

A A sudden onset of scrotal pain results from twisting of the spermatic cord. This results in cessation of blood flow and, ultimately, tissue necrosis if not diagnosed and treated rapidly.

After a tibial plateau fracture has been splinted, the nurse should be MOST concerned if the patient has? A) delayed capillary refill to the toes. B) swelling at the fracture site. C) increasing ecchymosis distal to the fracture. D) pain in the affected leg.

A A tibial plateau fracture extends into the knee joint. Joint involvement may be complicated by neurovascular compromise and associated fractures. With any fracture or dislocation, neurovascular status should be assessed before and after immobilization. If neurovascular status is compromised, further reassessment and interventions are required.

A patient has a regular, narrow complex tachycardia with a heart rate of 200 beats/min and a blood pressure of 90/60 mm Hg. An IV of D5W has been initiated and adenosine (Adenocard), 6 mg IV, is ordered. The nurse should? A) administer the adenosine undiluted in the port closest to the insertion site. B) reconstitute the adenosine with 3 mL sterile water or saline and administer as rapidly as possible. C) reconstitute the adenosine with 5 mL of D5W and administer over 1 minute. D) change the IV to normal saline solution and administer the adenosine.

A Adenosine (Adenocard) has a very short half-life. It must be given by IV administration in a large vein over 1 second, followed by a 20 mL saline flush and immediate elevation of the arm to enhance delivery to the heart.

A patient is diagnosed with a traumatic intracerebral hemorrhage. The patient's current vital signs are heart rate of 83 beats per minute, blood pressure of 82/40 mmHg, and ventilations are being assisted via bag-mask device. The nurse anticipates the administration of? A) 3% sodium chloride B) labetalol (Trandate) C) mannitol (Osmitrol) D) nicardipine (Cardene)

A Administration of 3% sodium chloride will help to decrease the patient's ICP without concern for diuresis. Labetalol and nicardipine are not clinically indicated due to the hypotension. Mannitol is to be avoided due to its potential diuretic effects and potential hypotension

A patient presents with a shoulder dislocation after a fall. The patient smells of alcohol, has slightly slurred speech, and answers questions appropriately. The nurse should prepare to ? A) obtain the patient's consent for a shoulder reduction. B) collect a blood alcohol level. C) monitor peripheral pulses until signs of intoxication are absent. D) contact the patient's family.

A Alcohol itself does not render a patient incompetent to give informed consent. The emergency clinician should evaluate each situation individually to determine whether the patient is incapacitated to the extent that they are unable to understand the proposed treatment, risks and benefits, and rational alternatives. In a healthcare setting, measured blood alcohol concentration does not necessarily indicate a person's ability to comprehend. Chronic alcoholics can be entirely rational and competent with a high blood alcohol concentration. Monitoring circulation in the affected extremity should continue but is not definitive care. The patient is not a minor, is able to answer questions appropriately, has no documented mental defects, and is able to understand risks and benefits of the procedure, therefore a legal guardian is not required for decisions to be made.

A patient with a history of hepatocellular carcinoma presents with jaundice, confusion and is combative. The nurse suspects? A) an elevated ammonia level. B) an elevated glucose level. C) elevated liver function levels. D) an elevated bilirubin level.

A Ammonia is a byproduct of protein metabolism, which is converted to urea by the liver and excreted by the kidneys. At high levels, ammonia crosses the blood-brain barrier and can lead to cerebral edema. Clinical features of hyperammonemia include sleep-wake cycle disturbances, irritability and confusion. As ammonia rises, it can lead to seizures, coma and eventually death. Elevated bilirubin, liver enzymes and hyperglycemia are significant findings but do not typically cause acute confusion.

After an occlusive dressing has been applied to a patient's open chest wound, the nurse should assess for? A) increased work of breathing B) tracheal deviation toward the injured side. C) dullness on the injured side.. D) stridorous respirations.

A An open chest wound can quickly become a tension pneumothorax if pressure builds inside the chest after an occlusive dressing has been placed. The patient will most likely exhibit increasing pain and dyspnea. The trachea will deviate AWAY from the affected side and the injured side will have hyperresonance.

Which of the following medications should a nurse anticipate administering initially to an adult patient in status epilepticus? A) diazepam (Valium) B) fosphenytoin (Cerebyx) C) flumazenil (Romazicon) D) phenobarbital

A Diazepam (Valium) is a benzodiazepine used as a first-line drug in the treatment of status epilepticus. Fosphenytoin (Cerebyx) is the second-line drug for seizure treatment after a benzodiazepine. Flumazenil (Romazicon) is a reversal agent for benzodiazepine overdose and can precipitate seizures. Phenobarbital is a long-term treatment to prevent seizure recurrence and is not a first-line medication for status epilepticus.REFERENCEEmergency Nursing Core Curriculum, 7th Ed. (2018), p. 360

A patient presents with a shoulder dislocation. The nurse knows the PRIORITY intervention is to? A) reduce as soon as possible. B) apply a shoulder immobilizer. C) prepare the patient for surgery. D) promptly consult an orthopedic physician.

A Dislocations are considered an emergency because of the danger to nearby nerves and blood vessels in the form of compression, stretching, or ischemia.

The nurse is assessing an alert patient who was trapped in a house fire. The patient is coughing, speaking with a hoarse voice and the nurse notes singed nasal hairs. BP 147/89 mmHg HR 117 beats/minRR 26 breaths/minO2 Sat 99% (room air) The nurse should anticipate the PRIORITY intervention being? A) endotracheal intubation B) chest radiograph C) breathing treatment D) arterial blood gas

A Early intubation is the priority in a patient with signs of acute inhalation injury. The signs include a hoarse voice, cough, black-tinged sputum and singed nasal hairs or eyebrows. The patient will have a chest radiograph and an arterial blood gas after intubation but it is not a priority over intubation. A breathing treatment may be indicated but airway protection takes priority over this intervention.

A patient who has ingested antifreeze is undergoing treatment. An indication that further treatment is needed is? A) metabolic acidosis. B) pupillary constriction. C) hyperreflexia. D) rotary nystagmus.

A Ethylene glycol is metabolized to produce glycolic acid that results in metabolic acidosis.

Which of the following statements by a patient with shingles indicates that discharge teaching has been effective? A) I can return to work when my lesions are crusted. B) I should use cortisone cream on my lesions. C) I should take all of my antibiotics. D) I should launder all my bedding in hot water.

A Fluid from the lesions is contagious. Advise the patient that they need to avoid work until there is no more fluid in the lesions and they are crusted over.

A pediatric patient is treated for otitis media complicated by a ruptured tympanic membrane. Discharge education is proven effective when the patient's father states? A) I will keep my son from getting his ear wet. B) I will use antibiotic ear drops four times a day for one week. C) I will position my son upright in bed when he goes to sleep. D) I will feed him soft foods until the ear drum heals itself.

A Fluid should be prevented from damaging the structures normally protected by the eardrum. Water provides an environment conducive to bacterial and fungal growth.

A patient who sustained major trauma has a blood pressure of 84/56 mm Hg, a heart rate of 124 beats/min, and respirations of 42 breaths/min. Breath sounds are absent on the left side, the trachea is deviated to the right, and the left thigh is deformed. The nurse should FIRST prepare to assist with? A) needle decompression. B ) pericardiocentesis. C) chest tube insertion. D) application of a traction splint.

A Following the ABC of assessment, the priority for this particular patient is breathing. This patient's respiratory status is compromised due to a tension pneumothorax for which immediate needle decompression is indicated.

The nurse is assessing a patient complaining of severe muscle cramps and states they recently started taking furosemide (Lasix). The nurse suspects? A) hypokalemia B) hyperglycemia C) hypercalcemia D) hyponatremia

A Furosemide(Lasix) is a loop diuretic that increases the amount of potassium that is excreted though the kidneys. Hypokalemia commonly causes muscle cramps where as hypercalcemia causes muscle weakness. Hyponatremia and hyperglycemia would cause general weakness and confusion but not muscle cramps.

Which spinal cord injury is characterized by greater loss of motor function in the upper extremities than in the lower extremities? A) central-cord syndrome B) Brown-Sequard syndrome C) anterior-cord syndrome D) posterior-cord syndrome

A Greater loss of motor function in the upper rather than the lower extremities are associated with central-cord syndrome.

A patient presents to the emergency department after engaging in an altercation involving a gun. After lifesaving interventions have been performed the nurse should? A) Place paper bags on the patient's hands. B) Label wounds as entrance or exit wounds. C) Place clothes in a plastic bag. D) Give family religious belongs that the patient was wearing.

A Hands should be wrapped in paper bags to preserve possible gun residue. Clothing should be placed in paper bags, not plastic, to prevent the growth of mold. Wounds should not be labeled as entrance or exit wounds by staff. All items with the patient at the time they presented to the ER should be properly given to the police and not the family as it is considered evidence.

A patient is diagnosed with hepatitis A. Which of the following statements indicates this patient has a good understanding of the discharge instructions? A) I understand that my close contacts may need treatment. B) I need to avoid alcohol for 6 months.. C) I need to work on rebuilding my strength. D) I understand transmission is from my sexual partner.

A Hepatitis A virus is transmitted by the fecal-oral route and is infectious 2 weeks before and 1 week after jaundice. Others who have been in close contact with the patient may need treatment. Transmission in this case may not have come from the sexual partner. Alcohol is contraindicated during the acute phase and rebuilding strength is not the best choice.

A nurse is assessing a patient who is tachycardic and tachypneic. The patient has recently traveled cross country on an airplane. The nurse should be MOST concerned that the patient may have? A) a pulmonary embolism B) pneumonia C) a pleural effusion D) a pneumothorax

A In a patient with a pulmonary embolism tachypnea and tachycardia are the most common findings. Immobility and a prolonged seated position are risk factors for developing a pulmonary embolism.

A child from a neighboring native American reservation presents with their caretaker who reports that the patient has been complaining of a sore throat for the last two days. During the assessment, the nurse notes that the patient is avoiding eye contact, looks down when spoken to, and defers to the caretaker to answer questions. The nurse understands this is likely? A) a normal cultural finding. B) human trafficking. C) a developmental delay. D) child maltreatment.

A In the Native American Culture, making direct eye contact is considered disrespectful. Children are raised to avoid eye contact and gaze downward when being spoken to by an adult or anyone with authority. It is also normal within this culture for the caregiver to answer questions for the child.

Which of the following is the MOST effective method to reduce the transmission of tuberculosis in the emergency department? A) negative-pressure room B) ultraviolet radiation C) use of paper surgical masks D) handwashing

A Infection is transmissible from person to person by aerosolized pulmonary secretions. Infected patients should be isolated, preferably in a negative-pressure room.

A patient who has periorbital bruising and cheek flattening after being kicked by a horse has an unsteady gait on arrival at the emergency department. The nurse should FIRST? A) place the patient in spinal precautions. B) order radiographs of the face. C) palpate the infraorbital rim. D) evaluate visual acuity and pupillary response.

A Initial care of a trauma patient includes considering the mechanisms of injury and following the Primary assessment, which consists of: A is for airway AND cervical spine B is for breathing C is the circulation D is for disability (perform a neurological assessment) E is for expose and environment The nurse must assume a cervical spine injury until proven otherwise. Radiographs will be done, BUT are not the first step in the care of this patient. Palpation of the face will be done during the Secondary assessment. Visual acuity will be done as a part of the Secondary assessment. Pupillary response may be done during "D" in the Primary assessment, but this is after cervical spine precautions have been initiated.

The nurse is assessing a patient who is anxious, hyperventilating and complaining of paresthesia in their fingers. The nurse knows that an appropriate INITIAL intervention is to? A) demonstrate breathing control techniques. B) ask about events leading up to their symptoms. C) instruct them to blow into a paper bag. D) insert an intravenous catheter.

A Initially the nurse should instruct the patient on deep breathing and muscle relaxation using therapeutic communication. Asking the patient about the precipitating event is not useful until the patient is calm and can think clearly. The patient may require an intravenous catheter for further treatment later in their care but this is not the initial intervention. Blowing into a paper bag for rebreathing has been associated in patient death due to hypoxemia complications.

A 6-month-old presents to the emergency department with report from the parents of intermittent abdominal distension and bloody mucus in the stools. The nurse suspects? A) intussusception. B) pyloric stenosis. C) rotavirus. D) gastroenteritis.

A Intussusception is the telescoping of one segment of the bowel into another and is most common in children between 3 months and 5 years of age. Intermittent abdominal distention and bloody mucus are signs and symptoms of intussusception.

A 175-lb (80-kg) adult patient who sustained electrical burns is administered an IV solution of lactated Ringer's containing sodium bicarbonate. Fluid administration is deemed adequate if the patient's? A) urine output is 75 to 100 mL/h. B) serum pH is 7.25. C) cardiac monitor shows only rare premature ventricular contractions. D) urine is pink-tinged.

A It is important to closely monitor urine output on all burn patients. This is especially true in the electrical burn patient, as the full extent of the burn injury may not be known. Urine output should be maintained (greater and equal to) 1 mL/kg/h. The normal serum pH is 7.35-7.45. Rare unifocal PVCs occur even in patients who have not sustained burns.

A patient presents with his family members, who state that he has struck his brother and has been violent and paranoid. The nurse should FIRST? A) place the patient into a treatment room with the door open. B) assess the brother for possible injuries. C) notify police of the assault. D) escort the patient to the quiet room to await the social worker.

A It will be easier to keep the patient calm and contained by moving him into a treatment room.

Polystyrene sulfonate resin (Kayexalate) is ordered for a patient who has a serum potassium level of 8.6 mEq/L because the drug? A) exchanges sodium for potassium ions. B) binds potassium to albumin. C) increases renal excretion of potassium. D) forces potassium into the cells.

A Kayexalate is used in the treatment of hyperkalemia because it is a cation exchange resin. Given orally or rectally, the resin is combined with potassium in the colon. The potassium is eliminated in the body through the feces.

Which of the following symptoms is MOST characteristic of early left ventricular failure? A) nocturnal dyspnea B) peripheral edema C) expiratory wheezing D) jugular venous distention

A Left ventricular failure causes blood to back up into the lungs. This can produce paroxysmal nocturnal dyspnea. The other findings listed are symptoms of right sided heart failure. In LATE left ventricular failure, the right heart will also fail and will then produce peripheral edema, jugular vein distention, and expiratory wheezing (cardiac asthma).

Neurogenic shock is MOST likely indicated if a patient has a blood pressure of? A) 86/50 mm Hg and a heart rate of 52 beats/min. B) 82/48 mm Hg and a heart rate of 125 beats/min. C) 100/80 mm Hg and a heart rate of 146 beats/min. D) 120/74 mm Hg and a heart rate of 50 beats/min.

A Neurogenic shock is a result of injury to the medullary vasomotor center or to the spinal cord. Resultant block of the sympathetic nervous system leaves the parasympathetic nervous system unopposed. Signs and symptoms of neurogenic shock include decreased blood pressure with associated bradycardia.

The nurse is caring for a young adult experiencing delusions, hallucinations, and thought disorganization. After medical clearance, the nurse suspects the patient is suffering from? A) Schizophrenia B) Bipolar Disorder C) Obsessive Compulsive Disorder D) Anxiety

A Onset of schizophrenia is typically in the young adult. Some signs and symptoms include hallucinations, delusions, thought disorganization, bizarre behavior, perseveration, ideas of reference, and ambivalence. Bipolar disorder is characterized by acute mania and severe depression. Obsessive Compulsion Disorder often begins during childhood or adolescence and generally involves obsessions and compulsions. Anxiety is a diffuse, unfocused response that alerts an individual to an impending threat; real or imagined.

A worker, pulled from inside a burning warehouse, is brought to the emergency department by bystanders. The first PRIORITY action for the emergency nurse is to? A) apply oxygen via nonrebreathing mask B) collect an ABG C) establish IV access D) attach the patient to the cardiac monitor

A Patients involved in indoor fires are at risk for cyanide exposure. Cyanide inhibits aerobic metabolism, which results in cellular death due to hypoxia, therefore 100% oxygen via the nonrebreathing mask is needed to supplement and prevent lactic acid build up. IV access, an ABG and cardiac monitoring are important, but are not the first priority.

The nurse is caring for a patient who has sustained a femur fracture. The patient suddenly has alterations in their mental status and a petechial rash is noted to the chest and axilla. The nurse immediately? A) administers oxygen B) prepares for chest tube insertion C) releases the traction splint D) prepares an intra-compartmental pressure monitor

A Petechial rash on the chest and axilla are associated with fatty embolism, which the patient would be at a higher risk for after sustaining a long bone fracture. The hallmark sign of fatty embolism is hypoxemia and altered mental status. Initial treatment is focused on symptom management and correcting hypoxia and hypotension. There is no indication to release traction or prepare for chest tube insertion at this time. An intra-compartmental pressure monitor is indicated for the detection of compartment syndrome, of which there are no symptoms indicated with the information provided.

A patient has recently been diagnosed with pulmonary hypertension. The emergency nurse anticipates which medication class will be found on reconciliation of their home medications? A) phosphodiesterase-5 Inhibitors B) angiotensin-converting enzyme inhibitors C) poly-ADP ribose polymerase D) angiotensin II receptor blockers

A Phosphodiesterase-5 Inhibitors are commonly used to treat pulmonary hypertension by causing pulmonary vasodilation this increases blood flow to the lungs and lowers blood pressure. Angiotensin II receptor blockers (ARBs) and Angiotensin-converting enzyme (ACE) inhibitors are used to treat cardiac hypertension. Poly-ADP ribose polymerase is a class of medication used to treat cancers.

A patient who is 15 weeks pregnant presents with light vaginal spotting and lower abdominal pain. What past medical history diagnosis increases the patient's risk of an ectopic pregnancy? A) Pelvic inflammatory disease B) Contraceptive use C) Previous pregnancy of twins D) Preeclampsia

A Scaring from a past infection, such as pelvic inflammatory disease, in the fallopian tubes can be a cause of ectopic pregnancy. Previous pregnancies and contraceptives used in the past do not affect risk of ectopic pregnancy. Preeclampsia is a late gestational complication, while ectopic pregnancies are first trimester, complications.

A 38 week pregnant patient involved in a motor vehicle collision presents experiencing frequent contractions. The FIRST step the nurse should take is to? A) assess airway patency while maintaining cervical spine immobilization. B) place the patient on continuous fetal heart rate monitoring. C) establish a peripheral IV and start IV fluids. D) check for crowning or other signs of imminent delivery.

A Stabilization of the mother is the best way to ensure survival of the fetus. The steps of the trauma assessment remain the same to ensure the stability of the mother. The other interventions are appropriate, but do not supersede the initial trauma survey that needs to take place first.

The FIRST intervention for an infant with complete airway obstruction after inhalation of a foreign body is? A) back blows. B) intubation. C) a finger sweep. D) abdominal thrusts.

A The American Heart Association recommends back blows followed by chest compressions to dislodge an obstruction. Abdominal thrusts should never be performed on an infant nor should a blind finger sweep be used. Intubation takes time and should not be the first intervention.

A 14-year-old patient whose airway is obstructed is conscious and unable to speak. The nurse should FIRST? A) administer abdominal thrusts B) remove the obstruction with Magill forceps. C) encourage the patient to cough. D) administer back blows.

A The American Heart Association recommends performing abdominal thrusts for a conscious choking patient; this should generate enough pressure to force the foreign body out of the trachea. Back blows are reserved for infant choking victims, and, without proper visualization, the Magill forceps are likely to force the blockage deeper. If the patient is obstructed, there will not be an effective cough.

What is the PRIORITY intervention for a patient who presents with the following vital signs? BP 65/42 mm HgHR 145 beats/minRR 6 breaths/min T (rectal) 102˚F (38.8˚C) O2 sat 85% on room air A) Start manual ventilations. B) Apply pads to chest. C) Prepare for intubation. D) Establish IV access.

A The first priority should be opening the airway and providing ventilatory support.

A patient presents with a nail in the eye after a nail gun accident at work. The nurse should? A) ensure the foreign object remains stabilized. B) profusely irrigate the affected eye. C) remove the nail to prevent further loss of vision. D) apply a pressure dressing over the affected eye.

A The foreign object should not be removed and irrigation is not appropriate in this scenario. Pressure dressings should be avoided to protect the globe. The foreign object should be kept as stable as possible.

An adolescent patient was killed in an MVC on the way home from school. Upon informing the grandmother who is the primary caretaker, the nurse observes moaning and diminished, slow speech. The PRIORITY action is? A) Provide privacy and supportive dialogue B) Complete mental health screening C) Initiate stroke protocol D) Check glucose levels

A The grandmother is showing signs of acute grief reaction which can manifest as changes in speech patterns and emotional lability. Privacy to grieve, supportive dialogue, and encouraging expressing of emotions are therapeutic interventions for acute grief reactions.

A patient is being discharged with a presumptive diagnosis of primary herpes simplex viral infection. The patient understands discharge instructions if she states? A) If I get pregnant, I can pass the disease to my baby. B) Acyclovir will cure my herpes. C) My boyfriend will not get this if he uses a condom. D) I know that this first episode will last only 4 or 5 days.

A The greatest risk to the fetus occurs when overt HSV infection is present at the time of delivery. In this situation, the principal mechanism of infection is direct contact with the infected vesicles during the process of vaginal birth.

Which of the following signs and symptoms are characteristic of pertussis? A) paroxysmal coughing and fever B) inspiratory stridor and drooling C) pleuritic chest pain and chills D) vocal changes and difficulty swallowing

A The initial stage of pertussis may be indistinguishable from the common cold. In the second stage of pertussis, the patient experiences fever and unremitting paroxysmal bursts of coughing ending with a high-pitched "whoop." Inspiratory stridor and drooling are characteristic of epiglottitis or retropharyngeal abscess. Vocal changes and difficulty swallowing are nonspecific findings suggestive of a variety of conditions including laryngitis, pharyngitis, peritonsillar abscess, and foreign body. Pleuritic chest pain and fever are commonly found in pleurisy.

Myocardial infarction is MOST likely to be fatal? A) during the first 2 hours after the onset of the chest pain. B) 1 to 2 weeks after the onset of the chest pain. C) 6 to 12 hours after the onset of the chest pain. D) 2 to 5 days after the onset of the chest pain.

A The most common cause of death following myocardial infarction (MI) is a fatal dysrhythmia, ventricular fibrillation, or ventricular tachycardia. These rhythms tend to occur shortly after coronary vessel occlusion as a result of irritation to the myocardium and the heart's electrical conduction system. Death hours or days post-MI are generally the result of cardiogenic shock or reinfarction. Cardiac deaths that occur weeks or months following an MI are largely the result of heart failure.

The nurse is about to administer nitroglycerin when the patient tells the nurse they took sildenafil (Viagra) earlier today. The nurse's IMMEDIATE action should be to? A) notify the physician. B) notify the pharmacy. C) administer the nitroglycerin. D) administer half the dose.

A The ordering physician should be notified if a patient has taken a phosphodiesterase inhibitor in the past 24 hours as nitroglycerin administration can result in severe hypotension. The nurse and the pharmacist cannot administer a different dose than what is ordered by the physician.

A patient is brought into the ED by coworkers after reporting feeling ill at work. After further evaluation, the patient also complains of night sweats, fever, and hemoptysis. The nurse should? A) ensure the patient is placed in a negative pressure room B) ensure the patient is decontaminated immediately. C) anticipate the administration of DuoDote (atropine/pralidoxime). D) anticipate the administration of hydroxocobalamin (Cyanokit).

A The patient is exhibiting signs of tuberculosis. The patient should be placed on airborne precautions, which include a negative pressure room when possible. Decontamination is not indicated at this time. Cyanokit is indicated for suspected cyanide poisoning. DuoDote is indicated for organophosphate toxicity.

The nurse is caring for a patient who became unresponsive after diving head-first into a shallow pool. The patient is experiencing refractory hypotension. The nurse should prepare for the administration of? A) norepinephrine. B) epinephrine. C) lactated Ringers. D) normal saline.

A The patient is most likely experiencing neurogenic shock after diving headfirst into a shallow pool. Norepinephrine would be indicated to provide vasoconstriction to improve the patient's blood pressure since the patient has a loss of vascular tone and therefore refractory hypotension. Because hypotension can lead to hypoperfusion and secondary spinal cord ischemia, prolonged severe hypotension (systolic blood pressure < 70 mm Hg) should be prevented and treated. Fluid resuscitation is often ineffective in such patients and may result in fluid overload. Thus, when there is persistent hypotension despite fluids, it is recommended to use vasopressor support with norepinephrine to be started at 0.05 µg/kg/min and titrated upward to a maximum dose of 1 µg/kg/min to achieve an MAP of 85 mm Hg.

A patient presenting with headache, fever and a stiff neck now complains of restlessness and is hypotensive with altered mental status. The PRIORITY intervention is to administer? A) antibiotics. B) antipyretics. C)analgesics. D) antiemetics.

A The patient should be treated for suspected bacterial meningitis, which requires urgent administration of antibiotics as patients can become severely ill and progress to septic shock. Analgesics, antipyretics and antiemetics may be used to treat the patient's symptoms but do not treat the underlying bacterial infection.

A heating/ventilation/air conditioning (HVAC) technician was brought to the Emergency Department after being found unresponsive in the newly-constructed house in which he was working. The patient is now lethargic, flushed, and ST-segment depression is noted on a 12-lead ECG. Vitals signs:BP 125/82mmHgHR 90 beats/minRR 22 breaths/minSpO2 98% (2L nasal canula) The nurse should? A) place a non-rebreathing mask on patient. B) administer naloxone (Narcan). C) prepare for immediate rapid sequence intubation D) prepare patient for the cardiac catheterization lab.

A The patient working on HVAC equipment in an enclosed space and now presenting with these symptoms (lethargy, flushing) should lead the nurse to highly suspect carbon monoxide poisoning. The priority is to provide the patient with 100% oxygen. Carbon monoxide has a higher affinity for hemoglobin than oxygen, and therefore displaces oxygen from hemoglobin which ultimately prevents oxygenation of tissues and organs. SpO2 monitoring is not reliable to determine oxygenation. This patient is lethargic but does not require intubation at this time. The patient is not showing any signs of opioid overdose therefore naloxone (Narcan) is not indicated. ST-segment depression in this case is the result of global myocardial hypoxia, not an acute myocardial infarction.

The nurse is caring for a patient, with history of COPD, who is exhibiting moderate respiratory distress. The arterial blood gas reveals: pH 7.10PaCO2 70mmHgPaO2 80HCO3- 24mEq/l The nurse should prepare the patient for? A) Bilevel positive airway pressure B) high flow nasal cannula C) non-rebreather mask D)rapid sequence intubation

A The patient's arterial blood gas is demonstrating respiratory acidosis (high PaCO2, low pH). First line treatment for hypercapnia in this instance would be bilevel positive airway pressure (BiPAP). Intubation is not usually favored as first choice in patients with COPD experiencing moderate distress. High flow nasal cannula and non-rebreather mask would only provide the patient with higher levels of oxygen, which is not indicated at this time.

A patient presents with a history of abdominal pain, nausea, and vomiting for 2 days. Upon standing, the patient complains of dizziness. The nurse should anticipate an order to do which of the following FIRST? A) Initiate IV access B) Insert a nasogastric tube. C) Draw blood for a CBC. D) Administer pain medication.

A The patient's symptoms suggest postural hypotension that is most likely secondary to hypovolemia. This is treated with IV fluid bolus. Laboratory values can be obtained to evaluate the patient's status, but the priority intervention should be fluid replacement. Pain medications and a nasogastric tube may be necessary, but not until the fluid deficit is addressed.

Which of the following patients should receive the HIGHEST priority of care? A) a confused and unkempt patient who is brought in by the police B) a diabetic patient with a lacerated finger C) a patient without previous health problems complaining of lower back pain D) a patient whose medical history is unremarkable, but complains of flank pain

A The presentation of the unkempt patient brought in by police describes a situation in which little is known about the patient's status. Because the patient is confused, an altered mental status is evident. Patients with altered mental states take priority over patients without life-threatening airway, breathing, or circulation problems.

The nurse is caring for a patient with chronic pancreatitis who is complaining of abdominal pain, nausea and vomiting. The abnormal labs anticipated are? A) hyperglycemia and hypocalcemia. B) hyperglycemia and hypercalcemia. C) hypoglycemia and hypocalcemia. D) hypoglycemia and hypercalcemia.

A The result of chronic pancreatitis is an inability to digest fats and carbohydrates properly. Insulin production is also affected, causing hyperglycemia. Hypocalcemia is often a lab finding due to hypoalbuminemia. The calcium level will need to be corrected if the ionized calcium level is low or in the presence of neuromuscular effects.

A patient presents to the emergency department with complaints of blindness in the right eye. The patient reports three similar episodes lasting a few seconds occurring over the last few days. The emergency nurse anticipates the patient to be experiencing which of the following? A) central retinal artery occlusion B) retinal detachment C) anterior uveitis D) acute angle-closure glaucoma

A The sudden unilateral loss of vision, known as central retinal artery occlusion, is caused by a thrombus or embolus. This condition may be preceded by moments of temporary visual loss, lasting seconds to minutes. Patients with anterior uveitis will experience "floaters" in the eye, redness, and pain. Patients with a retinal detachment will have visual field disturbances but not a temporary loss of vision. In acute angle-closure glaucoma, symptoms include blurred vision, halos around lights, eye redness and pain.

A patient admitted in hypertensive emergency is given a nicardipine (Cardene) drip. Treatment has been effective if the patient's mean arterial pressure decreases by? A) 20% to 25% over 1 to 2 hours. B) 10% to 15% over 30 to 60 minutes. C) 20% to 25% over 4 to 6 hours. D) 10% to 15% over 2 to 4 hours.

A The therapeutic intervention should be limiting the decrease in blood pressure to 25% within the first 2 hours of treatment. Rapid reduction of blood pressure slows the progression of end organ damage.

The nurse is discharging a patient diagnosed with esophagitis. Which statement would indicate further patient education is required prior to discharge? A) "I need to eat immediately before going to sleep." B) "I need to schedule a sleep study." C) "I need to avoid chocolate and coffee." D) "I need to quit smoking."

A Therapeutic interventions for esophagitis include: Assess airway and breathing (inflammatory response to an insult can cause compromise and can be a trigger for asthma or sleep apnea). Lifestyle modifications such as weight loss, avoiding foods that relax the lower esophageal sphincter, such as alcohol, chocolate, coffee, fatty foods, and eliminating smoking. Home measures to minimize reflux include elevating the head of the bed (e.g., 4 to 6 inches) and avoiding large volumes of food or drink, especially before bedtime. The patient stating they should eat right before bed needs further education and clarification prior to discharge home.

A patient presents to the emergency department with numbness and tingling of the right arm and right leg and expressive aphasia. Which of the following is the most important information to obtain? A) time of onset of symptoms B) history of atrial fibrillation C) recent use of aspirin products D) normal blood pressure for patient

A There is a very narrow window of opportunity for use of fibrinolytic therapy in stroke patients; it must be started within 4.5 hours of symptom onset.

The nurse is caring for a patient with altered mental status, poor oral intake, and diarrhea for several days. The patient's laboratory tests are as follows: pH 7.29HCO3 14 mEq/Lserum glucose level 380 mg/dLserum potassium level 3.0 mEq/L urinalysis positive for ketones. An order for regular insulin 10 units IV is received. The nurse should? A) hold the insulin and request an order for a potassium replacement first. B) administer the insulin, but request that D5 0.45 NS also be infused. C) hold the insulin and request an order for D5 0.45 NS to be given first. D) administer the insulin, but slow IV push over 5 minutes.

A This patient is already hypokalemic and the infusion of the insulin with further complicate this condition. The order should be held and clarified to receive potassium first. Dextrose-containing solutions (i.e. D5 0.45 NS) would only be indicated once the glucose is under 250 to prevent hypoglycemia. Slow IV push is another common route of administration, but has no relevance to this case.

A patient with schizophrenia presents to the ED after initiating antipsychotic medication therapy a few days ago. The patient is exhibiting diaphoresis, severe muscle rigidity, and altered mental status. Which order would the ED nurse question? The patient's vital signs are: BP: 175/101 mmHgHR: 122 beats/minRR: 24 breaths/minTemp: 103.1 FSpO2 91% (Room air) A) Administer epinephrine intramuscularly. B) Administer normal saline IV bolus. C) Administer dantrolene (Dantrium) IV. D) Administer oxygen via nasal cannula.

A This patient is exhibiting signs & symptoms of Neuroleptic Malignant Syndrome (NMS) caused by the prescribed antipsychotic medication. The nurse should question administering intramuscular epinephrine as this patient is not having an anaphylactic reaction. The other interventions are appropriate to stabilize the patient's condition.

The nurse is caring for a hemodynamically unstable patient who was involved in a motor vehicle collision. Which of the following criteria would indicate that tranexamic acid should be administered? A) unstable pelvic fracture with blood at the urinary meatus B) open fracture of the left lower extremity with bone exposed C) LeFort II fracture with trismus and pain D) bleeding scalp wound with a pressure dressing applied

A Tranexamic acid is not indicated for isolated head wounds and should be given within 3 hours of injury. With no obvious signs of uncontrolled bleeding, the open fracture is not an indication.

A patient with complete heart block is hypotensive and confused. The MOST appropriate treatment is? A) transcutaneous pacing. B) defibrillation. C) carotid massage. D) lidocaine (Xylocaine).

A Transcutaneous pacing is the recommended immediate intervention for a severely symptomatic patient in third-degree (complete) heart block. Other therapies (e.g., lidocaine, defibrillation, and carotid massage) would worsen this patient's condition.

Which type of breath sounds would MOST likely be heard in a patient who has acute pulmonary edema? A) diminished breath sounds B) bilateral crackles (rales) C) stridor D) pleuritic rub

B Increased fluids in the lungs cause crackles, wheezes, and productive cough with frothy white sputum that may have a pink tinge to it. Stridor is a sign of upper airway obstruction.

A trauma patient complains of shoulder pain, but has no history or evidence of shoulder injury. The patient should be examined for possible? A) injury to the cervical spine. B) visceral injury. C) pneumothorax. D) damage to the ulnar nerve.

B Injury to the spleen can manifest itself with pain that is referred to the left shoulder (Kehr's sign). A visceral injury may be the cause of the referred pain, secondary to stimulation of the phrenic nerve due to abdominal distention or irritation of the diaphragm

A patient complains of lower abdominal discomfort and is only able to urinate a small amount at a time. The patient's bladder is distended. What is the FIRST intervention that the nurse should take? A) Perform a bladder scan. B) Insert an indwelling urinary catheter C) Prepare the patient for CT scan. D) Obtain a urology consultation.

B In a patient with urinary retention, an indwelling urinary catheter should be inserted for immediate relief.

Which of the following arterial blood gas measurements is consistent with diabetic ketoacidosis? A) pH 7.26, PaCO2 55 mm Hg, HCO3 28 mEq/L B) pH 7.24, PaCO2 25 mm Hg, HCO3 12 mEq/L C) pH 7.51, PaCO2 32 mm Hg, HCO3 26 mEq/L D) pH 7.44, PaCO2 31 mm Hg, HCO3 23 mEq/L

B In diabetic ketoacidosis, the serum arterial or venous pH is less than 7.30. The PaCO2 is dropping as the body tries to compensate for the acidosis by blowing off the CO2 ; thereby, lowering that value and increasing the O2 (hyperventilating).

A patient presents to the emergency department stating that he wants a physical exam because his primary physician is not available. The nurse should? A) tell the patient to call his primary physician and make an appointment. B) direct the patient to sign into the emergency department C) tell the patient the the emergency department does not provide physical exams. D) give the patient information to a local urgent care.

B "Anyone presenting on hospital property requesting emergency medical treatment must receive an appropriate medical screening examination (MSE) to determine whether an emergency condition exists and, if so, must be stabilized prior to discharge or transfer"

Which of the following assessment findings associated with a skull fracture indicate the need for further intervention? A) rhinorrhea and headache or dizziness B) decreasing level of consciousness and restlessness C) early evidence of periorbital ecchymosis and photophobia D) otorrhea, nausea, and vomiting

B A change in the level of consciousness is an early sign of increased intracranial pressure.

An adult patient sustains blunt trauma to the abdomen in a motor vehicle crash. The patient refuses to allow any invasive treatments. Initial assessment reveals an ecchymotic area in the left upper quadrant. The nurse should? A) obtain consent from a relative for treatment. B) assess the patient's neurological status C) allow the patient to leave after signing a medical release form. D) request an order to sedate the patient.

B A rational, competent adult can refuse any treatment. The first step is to establish the patient's ability to understand the consequences of refusing treatment. Due to the mechanism of injury, the patient should be evaluated for the presence of a head injury.

The nurse understands that adenosine is administered? A) IV push over 1-2 minutes. B) IV push over 1-2 seconds. C) as a bolus over 10-15 minutes. D) as a bolus over 1 hour.

B Adenosine is administered undiluted 6 mg or less by rapid injection over 1-2 seconds.

A patient arrives to the ED stating she is 16 weeks pregnant with twins. The patient states she suddenly started to have bright red vaginal bleeding but has no pain. The nurse anticipates the patient will require? A) transfer to Labor & Delivery B) ultrasound examination C) abdominal MRI D) manual pelvic exam

B An ultrasound exam should be completed first to locate the placenta and rule out placenta previa. A manual pelvic exam could damage the placenta if placenta previa is present. The patient is still early in the pregnancy, so transfer to Labor and Delivery is not indicated. An ultrasound is the preferred radiological examination for this condition, not an MRI.

The MOST lethal method of anthrax exposure is by? A) contaminated water sources B) inhalation C) contact with open wounds D) ingestion of spores

B Anthrax can enter the body via several routes but the most lethal is via inhalation. It can start with symptoms of seasonal illness such as the flu but deterioration can occur rapidly and result in death within several hours with possible mediastinitis or hemorrhagic meningitis. Ingestion via food or water would eventually manifest as sepsis, abdominal pain, and bloody diarrhea. Skin contact would result in lesions itching and urticaria.

Cerebrospinal fluid found in the ears indicates? A) pharyngeal trauma. B) a basilar skull fracture. C) injury to the cervical spine. D) a subarachnoid hemorrhage.

B Basilar skull fractures occur in any of the five bones of the skull base but are most common in the temporal bone. Complications of skull fractures include infections, hematoma, cerebrospinal fluid leaks. Objective data includes CSF leak from the ears (otorrhea) or from the nose (rhinorrhea). There is no potential CSF leak with a subarachnoid hemorrhage, pharyngeal trauma or injury to the cervical spine.

The emergency nurse is treating a patient who was rescued from a burning building. The patient is restless, intermittently confused, and nauseated. The nurse understands the MOST likely cause of this presentation is? A) decreased carbon dioxide. B) increased carboxyhemoglobin. C) increased carbon dioxide D) decreased carboxyhemoglobin.

B Carbon monoxide poisoning is the most immediate threat to life in survivors of inhalation injury. Because carbon monoxide binds readily to hemoglobin, the most likely cause of the patient's symptoms is increased carboxyhemoglobin. The other options do not account for the patient's presentation.

A patient presents after a pit viper bite to the leg. The patient has sudden onset of bleeding from the nose and blood in his urine. Blood is oozing from the venipuncture and intravenous insertion sites. The nurse should expect? A) a decreased prothrombin time. B) a decreased platelet count. C) decreased fibrin split products. D) increased Factor XIII.

B Certain snake venoms will cause the patient to develop a decreased platelet count (DIC). DIC is the activation of the coagulation cascade that leads to excessive clotting and rapidly changing use and values of the clotting components including platelets.

A patient presents with a shoulder dislocation after being thrown from a motorcycle and striking a tree. The patient is alert and the airway intact. The HIGHEST priority intervention for this patient is: A) fluid resuscitation. B) cervical spine immobilization. C) pain management. D) shoulder reduction.

B Cervical spine immobilization is the highest priority following blunt trauma until the injury can be ruled out. The shoulder dislocation is a distracting injury that could mask injury to the cervical spine. Joint reduction is a high priority due to the risk of neurovascular compromise, but not greater than cervical immobilization. Pain management and fluid resuscitation, while part of this patient's care, are not the highest priorities.

Which of the following will be found in a patient with a severe asthma exacerbation? A) a loose and barking productive cough B) expiratory or absent wheezing C) prolonged inspiratory respirations D) infiltrates on the chest radiograph

B Clinical manifestations include cough, wheezing, prolonged expiration time, and reduced peak expiratory flow rate. Increased work of breathing and use of accessory muscles may also be present. Absent wheezing may be present when airways are so constricted that air movement is severely limited.

A man who briefly lost consciousness after a motor vehicle crash is alert and oriented upon arrival at the emergency department. The patient states that he is fine except for an abrasion where his head hit the door frame of the vehicle. The patient complains of blurred vision 1 hour later. The nurse should FIRST? A) obtain orthostatic vital signs. B) repeat the neurologic assessment. C) assess visual acuity. D) obtain blood glucose measurement.

B Considering this patient's mechanism of injury, a closed head injury should be suspected. Increased intracranial pressure due to edema or intracranial bleeding may be manifested by symptoms such as visual disturbances, decreased level of consciousness, or vomiting; therefore, a follow-up neurological assessment is warranted. There is no reason to suspect that this patient would have orthostatic changes in vital signs. Assessing visual acuity and obtaining a blood glucose measurement may be indicated if the patient's neurological status is unchanged, but neither of these would be the priority.

A patient presents with complaints of severe tingling and pain in the left leg and reports daily training for an upcoming marathon. The leg is cool, pale and the skin is taut. The PRIORITY is to? A) prepare the patient for an ultrasound. B) prepare the patient for an immediate fasciotomy. C) administer furosemide (Lasix) as ordered. D) administer IV fluids as ordered.

B Due to an increase in muscle mass, athletes are at risk for developing compartment syndrome. With the presenting signs and symptoms, the priority is to prepare the patient for a fasciotomy to prevent damage to the limb. An ultrasound may be helpful in ruling out a deep vein thrombosis (DVT); however, from the patient's history and presentation, it is unlikely the patient has a DVT. While the patient may require IV fluids, this would not be the initial treatment. Lasix is not indicated for this patient's clinical presentation.

A woman presents to the emergency department reporting pelvic pain, nausea, and vomiting. She states she is 2-weeks late for her menstrual cycle, and she had a positive pregnancy test the prior day. The physician is suspicious of an ectopic pregnancy with possible rupture. What is the PRIORITY intervention? A) Administer methotrexate (MTX) therapy. B) Prepare the patient for surgery. C) Initiate IV therapy with D5NS. D) Give RhoGAM if the mother is Rh positive.

B Ectopic pregnancy commonly present around the sixth week of gestation. Signs and symptoms can include pelvic pain, nausea/vomiting, positive pregnancy tests. Therapeutic interventions would include initiating IV therapy with lactated Ringer's solution or normal saline, administration of RhoGAM if the patient is Rh negative, and preparation for surgery if rupture is suspected. Methotrexate is only an option for those who are hemodynamically stable with no signs of bleeding

Bacterial meningitis is MOST strongly suggested by a fever and? A) confusion. B) nuchal rigidity. C) slurred speech. D) lateral nystagmus.

B Fever, stiff neck, and altered level of consciousness is the classic triad of symptoms associated with bacterial meningitis.

Results of laboratory studies for a patient with disseminated intravascular coagulation are MOST likely to reveal? A) decreased partial thromboplastin time. B) elevation of fibrin split products. C) decreased clotting time. D) normal fibrinogen levels.

B Fibrinogen levels are decreased due to excessive clotting in the vasculature. Clotting times are increased as clotting factors are used up by excessive clotting. As fibrin is broken down (fibrinolysis), the elevated level of circulating fibrin split products further interferes with platelet function.

Human tetanus immune globulin provides passive immunity for? A) 6 months. B) 30 days. C) 72 hours. D) 1 year.

B Human tetanus immune globulin provides passive immunity for 1 month.

What electrolyte imbalance is the most likely underlying cause for peaked T-waves shown on an electrocardiogram (ECG) tracing ? A) hypokalemia B) hyperkalemia C) hypermagnesemia D) hypercalcemia

B Hyperkalemia is characterized by an excess in serum potassium, exceeding 5.5 mEq/L. Effects of elevated serum potassium on the cardiovascular system include elevated and peaked T waves.

What is the MOST common cause of a hyphema? A) diabetic ketoacidosis B) traumatic injury C) diabetic ketoacidosis D) glaucoma

B Hyphemas are frequently related to trauma but certain medical conditions like sickle cell disease may also be the cause.

A patient arrives in cardiopulmonary arrest. The nurse notices a tattoo on the patient's chest that says "DNR." The nurse understand to? A) alert the resuscitative team of the patient's tattoo. B) continue advanced life support. C) stop all advanced life support efforts. D) initiate an emergent ethics consult.

B Nonstandard forms of DNAR (Do not attempt resuscitation) requests such as medical alert bracelets, tattoos and notes on prescription pads cannot be recognized as valid forms of advance directives. The emergency department team should continue advanced life support until legal documentation (e.g. Physician Orders for Life-Sustaining Therapy (POLST), durable medical power of attorney, written advance directives, state authorized no-CPR/DNAR directive), direction from the patient's legal decision-maker, or medical futility is reached.

A nebulized bronchodilator is administered to a 6-year-old child with severe asthma. Treatment may be deemed effective if the patient has? A) diminished breath sounds bilaterally. B) an increased peak expiratory flow rate. C) a respiratory rate that has decreased to 14 breaths/min. D) a prolonged expiratory phase.

B Peak expiratory flow rate can be used in children over the age of 5 and is used to monitor response to therapy for an acute episode of asthma, but is not designed to be a diagnostic too. The hallmark of asthma diagnosis is spirometry before and after bronchodilator therapy to document reversibility of airway narrowing, usually by peak expiratory flow rates.

A patient presents to the emergency department with chest pain that is worse during inspiration, a temperature of 100˚F (37.8˚ C) and a heart rate of 117 beats/min. The patient reports a myocardial infarction (MI) 4 days ago. The nurse should anticipate and order for? A) morphine sulfate B) aspirin C) furosemide (Lasix) D) antibiotics

B Pericarditis can occur following a myocardial infarction due to inflammation of the pericardial sac, leading to different treatment. Once an MI has been ruled out, aspirin should be administered for pain management. The first line of treatment for pericarditis is a non-steroidal anti-inflammatory drug (NSAID), not opioids. The patient does not have any signs of heart failure that would indicate Lasix administration. Low-grade fevers may be present due to the inflammation, but as this is not a bacterial cause of pericarditis, antibiotics are not indicated.

The emergency nurse knows that, to help prevent an intubated patient from aspirating, the PRIORITY intervention is ? A) preparing the patient for fiberoptic bronchoscopy. B) placement of an oral or nasal gastric tube. C) performing frequent assessments of all lung fields. D) placing the head of the bed at 30 degrees.

B Placement of an oral gastric tube to decompress the stomach helps prevent gastric distention and aspiration and should be placed after intubation. Performing a fiber optic bronchoscopy may be necessary to remove already aspirated blood or foreign debris, but is not a preventative intervention. Frequent assessment of lung sounds is helpful in the detection of possible aspiration, and placing the bed at 30 degrees is helpful in preventing aspiration but is not a priority intervention.

A patient's eyes are irrigated with 2 L of normal saline solution after being contaminated by plaster dust. Immediately following copious irrigation, the conjunctival pH is 7.0. This reading indicates that? A) the cornea has probably been damaged. B) treatment has been effective. C) continued irrigation is necessary. D) visual impairment may be permanent.

B Plaster is considered to be an alkali substance that requires copious irrigation due to the penetrating properties of alkalis. The normal pH of the eye is 7.0. The pH level indicates that the eye has returned to normal in this patient.

Which of the following statements regarding concussions indicates patient discharge teaching has been effective? A) I'll try to stay awake for the next 12 hours. B) I may have memory problems over the next year. C) I expect to be back to normal after 2 days. D) I should avoid solid food for the next 24 hours.

B Post-concussive syndrome may occur immediately after a head injury or may not appear until several days or months after the head trauma. Symptoms typically resolve within several weeks but have been known to persist for extended periods of time. Signs and symptoms may include a persistent headache, memory loss, impaired judgment, and irritability.

A patient who has a small puncture wound of the finger from a high-pressure paint gun has swelling in the hand. The nurse should FIRST? A) apply ice to the finger. B) prepare for immediate surgical intervention. C) prepare to administer local anesthetic infiltration. D) soak the finger in warm, sterile saline solution.

B Puncture wounds from a high-pressure injection may exhibit small entrance wounds, but the excessive force through the tissue planes, tendons, and associated structures may cause significant injury. Warm saline soaks may increase swelling and ischemia. Direct application of ice may cause a cold injury to the finger.

Hypovolemic patients who require massive transfusions should be given warmed blood because cold blood? A) increases oxygen-carrying capacity. B) increases the risk of myocardial dysrhythmias. C) exacerbates metabolic alkalosis. D) lacks calcium-binding citrate

B Rapid infusion of large amounts of cool or cold fluids results in iatrogenic hypothermia. The cardiovascular system is particularly sensitive to hypothermia. The cold heart becomes irritable. A variety of dysrhythmias may be seen.

A patient reports right elbow pain and swelling following sustained periods of repetitive motion. The MOST appropriate nursing intervention includes? A) preparation for incision and drainage. B) application of a compression dressing. C) administration of ordered antibiotics. D) administration of ordered steroids.

B Repetitive use of an extremity should lead the nurse to suspect tendonitis. Tendonitis is the inflammation of tendons and tendon muscle attachment. The appropriate intervention for tendonitis is to immobilize and support the affected area. Unlike the patient with bursitis, the patient with tendonitis has no fluid collection to drain.

Which of the following solutions is appropriate for INITIAL resuscitation of patients who have major burns? A) D5W B) lactated Ringer's C) 3% NS D) D5 0.45 NS

B Replacement of intravascular volume is the priority intervention for the patient with a thermal injury. Isotonic crystalloid solutions, such as lactated Ringer's or 0.9% normal saline, are recommended for fluid resuscitation.

A patient who has a serum potassium level of 2.6 mEq/L should be MOST closely monitored for? A) edema of the extremities. B) weakness of the respiratory muscles C) increased neuromuscular irritability. D) carpopedal spasm.

B Respiratory muscle weakness related to hypokalemia may lead to respiratory failure or death.

The ED nurse is caring for a patient with an aortic dissection. Vital signs are: HR: 52 BPMBP: 135/70 mmHgSPO2: 100% on 2L nasal canula The MOST appropriate infusion to initiate on this patient is? A) esmolol (Brevibloc) B) nicardipine (Cardene) C) epinephrine (Adrenaline) D) fentanyl (Sublimaze)

B The most important management for patient's with aortic injury is blood pressure management, with a goal of 90-120mmHg. Esmolol is not indicated for this patient due to bradycardia. Epinephrine will increase the blood pressure. Fentanyl is important for pain control, but will not affect blood pressure effectively for this patient.

The nurse is caring for a patient with an obvious femur fracture and bleeding from the urethra. The nurse should question which of the following orders? A) viscoelastrometric testing B) traction splint C) ankle-brachial index D) elevation of extremity

B The nurse should be suspicious of a possible pelvic injury with urethral bleeding. Traction is contraindicated in the presence of a pelvic injury. The other interventions are appropriate for this patient's injuries.

After providing 2 L of normal saline solution, initiating an insulin drip, and monitoring fingerstick blood glucose for the past 2 hours in a patient in diabetic ketoacidosis, what electrolyte would the nurse expect to be reassessed? A) sodium B) potassium C) magnesium D) calcium

B The nurse should expect to measure serum electrolytes at the time the patient arrives and every 2 to 4 hours afterwards, since fluid resuscitation, insulin therapy, and acidosis correction reduce extracellular potassium level.

A patient who sustains a head injury has increased pressure on the left oculomotor nerve. Assessment of the left eye is MOST likely to reveal? A) ciliary spasm of the eyelid. B) dilation of the pupil. C) conjunctival edema. D) ptosis of the eyelid.

B The oculomotor nerve (cranial nerve III) regulates pupil size and reactivity on the side ipsilateral to the injury.

Which of the following findings suggests that a patient may have a ruptured ovarian cyst? A) bloody vaginal discharge B) sharp abdominal pain C) thick yellow vaginal discharge D) intermittent abdominal pain

B The pain associated with a ruptured ovarian cyst is characterized as sharp and constant rather than intermittent. There is usually no associated vaginal discharge.

The Emergency Nurse placed an intravenous catheter and administered normal saline, later recognizing signs of infiltration at the site. The nurse followed all policies and procedures appropriately and identified the infiltration via assessment. Immediate action was taken to reduce the swelling, but the patient is now threatening to sue for malpractice. The ED Nurse understands that this case is missing an element of Nursing Malpractice because there was no: A) duty to the patient. B) breach of duty. C) damage incurred. D) proximate cause.

B The patient did experience damage, and the clear cause was the infiltration of the IV fluid. However, because the nurse followed all of the appropriate policies and procedures and acted as a prudent nurse would, there was no 'Breach of Duty'. Had the nurse NOT followed appropriate procedures, the case could be made for Nursing Malpractice.

A patient presents to the emergency room with complaints of a bull's-eye shaped rash after removing a tick. The patient also reports headaches, fatigue, malaise, and left-sided facial weakness. What diagnosis would the nurse expect? A) Cellulitis B) Lyme Disease C) Rocky Mountain spotted fever D) Tularemia

B The patient has a classic sign of Lyme Disease: bull's-eye shaped rash. Bell's Palsy can also be a symptom of Lyme Disease. This is not present in the other disease processes.

An unrestrained driver that is 8 months pregnant is involved in an MVC. They arrive complaining of shortness of breath and severe abdominal pain. Distended jugular veins are noted on assessment. The PRIORITY intervention is? A) Complete a bedside abdominal ultrasound B) Perform needle decompression in the second intercostal space C) Insert chest tube between the 4th and 5th intercostal space D) Place patient in the left lateral recumbent position

B The patient is displaying signs of a tension pneumothorax which requires immediate needle decompression. Placing patient on her left side would take pressure off of the inferior vena cava but would not be the priority action. Chest tube placement on a patient 8 months pregnant would be inserted between the 5th and 6th intercostal space due to displacement of the diaphragm by the gravid uterus. FAST exam limitations in pregnancy make it challenging to determine intrauterine versus extrauterine for a fluid assessment.

What is the most appropriate electrical therapy for the patient whose electrocardiogram (ECG) shows V. fib? A) external pacemaker B) defibrillation C) synchronized cardioversion D) transvenous pacemaker

B The rhythm strip shows a patient in ventricular fibrillation. In ventricular fibrillation the patient has no organized cardiac activity and no pulse. The early application of an unsynchronized DC countershock is the appropriate response. Pacemakers, external or transvenous, can pace a heart but will not terminate fibrillation. Synchronized countershocks are appropriate for a patient with a discernible R wave (e.g., a patient in supraventricular tachycardia) or a patient in ventricular tachycardia with a pulse.

The nurse is caring for an unresponsive, intubated patient who was found floating on top of the water in a pool. The event was unwitnessed. The ED nurse anticipates this is the highest priority? A) gastric tube insertion. B) cervical spine stabilization. C) fluid resuscitation. D) re-warming protocols.

B The unwitnessed injury and resulting patient status means c-spine stabilization is the highest priority as traumatic injury cannot be ruled out. Protecting the c-spine from further injury is the highest priority for this patient. Rewarming protocols, fluid resuscitation and gastric tube insertion are appropriate for this patient's treatment, however they are not higher priority than stabilizing the cervical spine.

A patient is being discharged with a new diagnosis of migraine headache. Which patient statement indicates successful discharge teaching? A) "I have an infection that caused the headache." B) "I will keep a headache journal to identify triggers." C) "I need to follow up with the neurosurgeon." D) "I should have a repeat CT scan in 2 days."

B There are no diagnostic tests to confirm the diagnosis of a migraine headache. Diagnosis is based on the patient's history and symptoms. Migraine headaches may be triggered by food, dieting, emotion, menses, medication, weather changes, sleep disturbances and bright lights. Patients should be encouraged to keep a headache journal in an attempt to identify triggers. A follow up with neurosurgery and concerns for infection are not recommended following this diagnosis.

A patient reports sexual assault. Vital signs are stable, no visible injury is noted, and the patient is able to clearly describe the events that took place. What acuity level should be assigned using the five-tier Emergency Severity Index (ESI) criteria? A) level 3 B) level 2 C) level 4 D) level 1

B These patients warrant a level 2 categorization because of the time-sensitive nature of the required treatment as well as the severity of their psychological pain.

A 4-year-old patient is brought to the emergency department the day after having a tonsillectomy. The patient is diaphoretic, tachycardic, and crying, and has obvious oral bleeding. The nurse should FIRST? place the patient on continuous cardiac monitoring. suction bloody secretions from the mouth. initiate an IV of lactated Ringer's solution at 50 mL/h. administer oxygen via nonrebreather mask.

B This child's airway is at risk. Following the ABC of assessment, the patient's airway needs to be cleared of blood through suctioning.

When providing care to a victim of sexual assault, the most important concept of trauma-informed care includes? A) compliance with protocols for victims of sexual assault. B) involving the patient in all decisions regarding treatment. C) ensuring all necessary consents are completed. D) recognizing injuries that will need to be treated.

B Trauma informed care includes the patient in all decisions regarding the recommended courses of treatment including medications, photography, and the physical exam. It focuses on the strength of the survivor and helps restore a sense of control.

The nurse is caring for a patient who ingested an entire bottle of their tricyclic antidepressant medication. The nurse would question which of the following orders? A) sodium bicarbonate B) ondansetron (Zofran) C) activated charcoal D) acetaminophen (Tylenol)

B Tricyclic antidepressant (TCA) toxicity can cause multiple ECG changes such as a widened QRS and a prolonged QT interval. Medications that can also cause a prolonged QT, such as ondansetron (Zofran), should be avoided. There is no contraindication for acetaminophen (Tylenol) at this time. Activated charcoal and sodium bicarbonate are frequently used in the treatment of TCA toxicity.

A patient with tuberculosis is considered infectious as long as? A) a productive cough is present. B) viable tubercle bacilli are discharged in the sputum C) the tuberculin skin test is positive. D) pulmonary tuberculosis is evident on the chest radiograph.

B Tuberculosis spreads when a person is exposed to tubercle bacilli carried in airborne particles or droplet nuclei. A patient is considered infectious until 14 days of directly observed therapy, 3 negative AFB smears, decrease in cough, and afebrile.

A patient who has a neck injury is to be moved. The nurse should protect the patient's cervical spine by? A) immobilizing the head by placing a hand over the forehead and chin. B ) stabilizing the head and neck in a neutral position. C) asking the patient not to move his head. D) placing the patient on a long backboard.

B When a cervical injury is suspected, the head should be manually maintained in a neutral position while a rigid cervical collar is placed. Simply placing the patient on a backboard or placing the hands over the chin and forehead will not effectively immobilize the cervical spine. Even the most compliant patient cannot prevent spontaneous head movement.

EMS arrives with a patient in severe respiratory distress and states the family reported he has not received dialysis in over two weeks. Which of the following medications should the nurse question? A) Midazolam (Versed) B) Succinylcholine (Anectine) C) Vecuronium (Nocuron) D) Fentanyl (Sublimaze)

B When succinylcholine is administered it causes an increase of extracellular potassium, and should be used cautiously in patients with suspected hyperkalemia. All other choices would not be contraindicated for this patient.

A patient with hypertension is complaining of "tearing" chest and back pain. The nurse should be MOST concerned that the patient is experiencing? A) a tension pneumothorax. B) acute aortic dissection. C) cardiac tamponade. D) a pulmonary embolism.

B With an acute aortic dissection, patients report sudden onset of excruciating pain described as sharp, ripping, tearing or knifelike. Initial treatment involves stabilizing the patient while safely reducing blood pressure.

A patient presents with a decreased level of consciousness and hypoxemia despite supplemental oxygen and has minimal air movement on auscultation. The nurse should anticipate? A) insertion of a chest tube. B) insertion of an endotracheal tube. C) administration of magnesium sulfate. D) initiation of continuous positive pressure ventilation.

B A decreasing level of consciousness, hypoxemia despite supplemental oxygen and a "silent chest" indicate minimal airflow and the need for endotracheal intubation. Magnesium sulfate is not indicated for this patient. The patient will not be able to tolerate continuous positive pressure ventilation if they are not alert and able to maintain an upright position. A chest tube will only be indicated if a pneumothorax is suspected.

Following an explosion at a chemical plant, a patient is found unresponsive and sustains multiple chemical burns from being unable to self extricate. These injuries were sustained in which phase of the blast? A) Tertiary B) Quaternary C) Primary D) Secondary

B Quaternary injuries are the final result of a blast mechanism, often due to the inability to extricate and continued injury occurring following the blast. All body systems are at risk for injury (i.e. thermal or radiation exposure and burns, chemical inhalation injuries). Primary injury is the initial impact of the blast wave contacting the patient (i.e. hollow organ injury). Secondary injury is impact associated with fragmentation or debris from the device or impacted objects (i.e. lacerations, and fractures). Tertiary injury is impact from the patient making contact with a surface following the force of the blast wave or flying debris (i.e. closed head injuries).

A patient who has chest pain and palpitations is given a dose of adenosine (Adenocard) by rapid IV push. The optimal outcome is? A) a transient decrease in heart rate. B) the chest pain decreases. C) the supraventricular tachycardia resolves. D) the systolic blood pressure increases.

C A decrease in chest pain and an improvement in systolic blood pressure are benefits that can be derived from adenosine therapy in a patient with supraventricular tachycardia (SVT). However, resolution of the tachydysrhythmia is the desired endpoint and indicates drug effectiveness. A transient decrease in heart rate following adenosine administration, even frank asystole, indicates the drug has reached the heart, but this is not the endpoint of therapy. Adenosine (Adenocard) should be given until conversion occurs or maximum dose is given.

Following a thoracentesis, a patient suddenly complains of worsening chest pain and shortness of breath. The nurse is MOST concerned for? A) pulmonary hypertension. B) pulmonary embolism. C) pneumothorax. D) pneumonia.

C A pneumothorax can occur after a thoracentesis due to air entering the pleural space, or puncture of the lung. The development of a pneumonia would not be cause for the sudden onset of worsening symptoms after the procedure. Other complications of a thoracentesis are pain and pulmonary edema, not pulmonary embolism or pulmonary hypertension.

Which finding would be MOST likely in a patient with an acute aortic dissection? A) decreased body core temperature B) increased body core temperature C) upper extremity blood pressure variation D) expiratory wheezing bilaterally

C A significant variation in blood pressure readings in each arm is indicative of an aortic dissection.

Which of the following statements about subdural hemorrhage is true? A) It results from a thrombosed artery. B) It results from a tear in the middle meningeal artery. C) It may initiate an acute or chronic onset of symptoms. D) It occurs between the skull and the dura.

C A subdural hemorrhage is usually due to venous bleeding that results from tearing of bridging veins. An acute subdural hemorrhage displays signs and symptoms within 48 hours after injury. A chronic subdural hemorrhage may not display signs or symptoms for 2 weeks or longer.

The emergency nurse is preparing to assist with a bedside thoracentesis for a patient with lung cancer. The nurse understands this procedure is to alleviate symptoms caused by? A) pulmonary fibrosis. B) pulmonary embolism. C) pleural effusion D) pneumonia.

C A thoracentesis is a procedure to remove fluid from the pleural space. This condition is more likely to occur in patients who have a lung tumor. A thoracentesis would not alleviate the symptoms of the other three diagnoses.

A 33-year-old patient with a history of sickle cell disease complains of a cough, low-grade temperature for the past two days, and chest pain that developed this morning. The nurse anticipates the PRIORITY for this patient is? A) administration of tissue plasminogen activator. B) administration of antipyretics C) a chest x-ray. D) blood cultures.

C Acute chest syndrome is the most common pulmonary disease associated with sickle cell disease and is the most common cause of death. Patients with acute chest syndrome have fever, cough, chest pain and new infiltrates found with chest radiograph. Antipyretics and blood cultures are not a priority intervention. tPA is not indicated for acute chest, as it is primarily a result of an infectious process.

A patient presents with the complaint of "something is in my eye." A piece of metal is removed and a rust ring is noted. Discharge teaching has been successful when the patient states? A) "I will need to rest my eyes for a week." B) "I will take antibiotics until the rust coloring goes away." C) "I will follow up with an ophthalmologist in 24 hours." D) "I will need to wear an eye patch for a few days."

C Additional rust rings can develop within 24 hours. Follow up is essential because fragments of metal could have been missed during removal, and other rust rings could develop, leading to infection and other complications.

Following the transfusion of a unit of packed red blood cells, a patient develops dyspnea, tachypnea, and posterior bilateral crackles. The nurse should anticipate an order for? A) calcium chloride. B) nesiritide (Natrecor). C) furosemide (Lasix). D) digoxin (Lanoxin).

C Adverse effects of blood transfusions include circulatory overload. Signs/symptoms include those similar to heart failure (dyspnea, tachypnea, crackles). Treatment includes administering diuretics (furosemide).

A patient with a history of alcoholism is vomiting bright red blood and has a blood pressure of 98/54 mmHg and a heart rate of 140 beats/min. After intubation, nurse should anticipate? A) administration of lorazepam (Ativan). B) placement of indwelling urinary catheter. C) initiation of mechanical tamponade. D) placement of a central line.

C After intubation, mechanical tamponade would be the priority intervention to stop the active bleeding and prevent hypovolemic shock due to esophageal varices. This can be accomplished with the insertion of an inflatable tube inserted into the esophagus. Administration of Ativan and placement of an indwelling urinary catheter are not priority interventions. Placement of a central line may be necessary to facilitate fluid and blood product administration but would not take precedence over stopping the bleeding.

What is a PRIORITY diagnostic test that should be performed when treating a patient with a suspected dysrhythmia? A) troponin level B) chest X-ray C) electrocardiogram D) CT scan of the chest

C An electrocardiogram is the first priority because it is necessary to know the type of dysrhythmia to determine treatment and causes.

The nurse is triaging a 3-week-old child whose mother reports the patient felt hot, has a lack of appetite, is not wanting to be held, and has red bumps to their skin. Triage vitals are HR: 168 beats/minRR: 54 breaths/minT: 101.8˚F rectal Bulging fontanels are observed. This patient will require rapid evaluation to rule out which disease? A) pertussis B) mononucleosis C) meningitis D) varicella

C Any infant younger than 2 months old with fever or subnormal temperature must be evaluated for meningitis. This patient is exhibiting the classic signs of meningitis including fever, poor feeding, disinclination to be held, and non-blanching rash. While pertussis, varicella and mononucleosis are all diseases which may effect this population, the most emergent condition requiring immediate evaluation is meningitis. The ED nurse should anticipate an entire work-up including blood cultures and a lumbar puncture.

A patient presents with complaints of feeling extremely dizzy, chest tightness, and states their "heart is racing". They report a history of anxiety with frequent panic attacks. The PRIORITY intervention is to? A) Request an order for anti-anxiety medication B) Apply supplemental oxygen C) Obtain an ECG D) Notify psychiatric services

C Because symptoms of a panic attack can mimic a coronary event, it is a priority to rule out out physiologic causes. Oxygen may be therapeutic to help the patient normalize their respirations as well as anti anxiety medications but would not help rule out physiologic causes. A psychiatric consult may be necessary once the patient is medically cleared.

In planning care for an infant in shock, a nurse should recognize that? A) rapid infusion of 7.5% sodium bicarbonate is essential to combat acidosis. B) hypothermia improves the infant's chance of survival. C) moderate volume loss may cause profound hypo-perfusion. D) tachypnea is the cardinal sign of impending shock.

C Blood volume in a child is 80 mL/kg, less than that of an adult and with a lower oxygen reserve. Volume loss can produce hypovolemia and hypoperfusion more quickly than in adults. Since stroke volume in the pediatric patient is relatively fixed, the child naturally compensates for a decreased cardiac output with tachycardia. Restoration of circulating volume with warmed fluids, blood, and blood products is the treatment of choice.

Which of the following is the BEST description of bronchiolitis? A) a bacterial infection with a sudden onset resulting in inflammation of the epiglottis B) a bacterial infection with a sudden onset accompanied by decreased breath sounds C) a viral infection with a slow onset causing lower respiratory tract inflammation D) a viral infection with a rapid onset accompanied by barking cough

C Bronchiolitis is a viral infection most commonly caused by respiratory syncytial virus, adenoviruses, and influenza. It characteristically affects the respiratory tract causing an inflammatory response that results in increased respiratory secretions. These secretions may cause obstruction in the smaller airways. Symptoms become progressively worse over a period of several days.

A patient with pancreatic cancer presents to the emergency department in acute distress with sudden shortness of breath, palpitations, anxiety and chest pain exacerbated with deep breaths. The patient has a history of recurrent bilateral deep vein thrombosis. The patient's vital signs are: BP 101/57 mm HgHR 117 beats/minRR 24 breaths/minT (oral) : 98.8˚ F (37.1˚ C)O2 sat 92% on room air The nurse suspects? A) a myocardial infarction. B) pericardial tamponade. C) a pulmonary embolism. D) a spontaneous pneumothorax.

C Cancer cells are prothrombic as malignant cells release procoagulants and cytokines, both factors frequently leading to thrombosis. Of all cancers, pancreatic and gastric cancer have the highest risk for thromboembolic disease. Based on the patient's medical history and presenting signs and symptoms, this patient is likely experiencing a pulmonary embolism. The other diagnoses are incongruent with the patient's medical history, signs and symptoms.

A patient complains of chest pain that worsens with inspiration or lying supine and lessens by leaning forward. The nurse should suspect? A) endocarditis. B) pleurisy. C) pericarditis. D) costochondritis.

C Chest pain that worsens with inspiration or lying supine and lessens by leaning forward is a classic description of pericarditis. Costochondritis worsens with manual pressure against the anterior chest wall. Pleuritic pain is aggravated by deep inspiration. Endocarditis is generally painless but is often associated with systemic illness.

Chronic arterial insufficiency of the lower extremities is characterized by? A) dilated tortuous veins. B) pitting edema. C) intermittent claudication D) bounding pedal pulses.

C Claudication is caused by poor tissue perfusion or ischemia due to gradual enlargement of atheromatous plaques

Assessment of a patient with a hemorrhagic stroke affecting the left cerebral hemisphere is likely to reveal? A) deviation of the head and eyes to the right. B) blindness in the left half of both visual fields. C) expressive, receptive, or global aphasia. D) left hemiparesis or hemiplegia.

C Clinical signs and symptoms of the patient depend on the size and location of the hemorrhage. Based on how our nervous system is set up an injury to one side of the brain generally affects the opposite side of the body. Left-sided hemorrhagic stroke patients may have aphasia or receptive aphasia if there is damage to language centers of the brain. For almost all right-handers and for about 1/2 of left-handers, damage to the left side of the brain causes aphasia. As a result, individuals who were previously able to communicate through speaking, listening, reading, and writing become more limited in their ability to do so.

Which of the following findings in a comatose patient who has diabetes mellitus suggests hypoglycemia rather than hyperglycemia? A) Kussmaul respirations B) hot, dry, flushed skin C) pale, clammy skin D) poor skin turgor

C Common signs of hypoglycemia include: weakness, dizziness, altered mental status, shallow/normal respirations, and pale, cool diaphoretic skin. Common signs of hyperglycemia include: altered mental status, deep and rapid respirations, hot and dry skin, and may be hypotensive.

A patient presents with a crush injury to the lower leg. The most concerning finding is? A) 3+ dorsalis pedis pulses B) debris in open wounds C) increasing pain despite analgesics. D) mild continued bleeding.

C Compartment syndrome is always a concern with crush injuries. A hallmark symptom of compartment syndrome is increasing pain that is unrelieved by analgesics--making this the priority assessment finding.

During quality review, the Quality Improvement Committee identifies a variance from the standard of nursing care of two patients with chest pain. Which of the following is the MOST appropriate recommendation by the Committee? A) Design a new flow sheet for cardiac documentation. B) Request the nurse manager to discipline the nurses involved. C) Conduct a random audit and determine the frequency of the problem. D) Schedule a nursing inservice on the standard of care for chest pain.

C Conducting a random audit is most appropriate in this scenario. The Quality Improvement Committee needs to determine whether these two cases are isolated incidents or if there is an ongoing deficiency in the standard of care delivered. Discipline, inservices, or new flow sheets are not considered until the scope of the problem is defined.

A patient presents to the Emergency Department in respiratory distress. The patient has a history of COPD and has an oxygen saturation of 89% on 4 liters of oxygen. The ED nurse understands this patient should receive oxygen delivery via? A) Continuous positive airway pressure B) High-flow nasal cannula at 100% FiO2 C) Bi-level non-invasive ventilation D) Non-rebreather mask at 100% FiO2

C First-line therapy for patients with acute COPD exacerbation with respiratory failure is Bi-level non-invasive ventilation. This decreases mortality, morbidity, and hospital length of stay. The other options are not the first-line therapy choice for this patient.

An essential requirement that needs to be proved in a medical negligence case is the presence of? A) failure to act. B) intent to damage. C) damages. D) incorrect action.

C Four elements of negligence that must be proven include duty, breach of duty, proximate cause, and damages. Incorrect actions or failure to act alone are not essential elements of negligence. Every negligence case must prove damages.

A patient is being treated for an acute exacerbation of chronic obstructive pulmonary disease. An improvement in the patient's condition is indicated if the patient is? A) able to cough up thick, yellow sputum. B) using pursed-lip breathing techniques. C) able to speak in uninterrupted sentences D) maintaining the tripod position.

C General assessment of a patient with a respiratory emergency includes speech pattern (e.g., How many words can a patient speak before needing to breathe or how many phrases or full sentences can the patient state?). Pursed-lip breathing and tripod position both suggest continued distress. Thick yellow sputum is not unexpected and neither suggests improvement nor deterioration.

A patient presents with vision changes to the right eye, fever, fatigue and a rash on the nose and forehead for two days. Extraocular movement is intact in the right eye. The nurse suspects? A) orbital cellulitis. B) ischemic stroke. C) herpes zoster ophthalmicus. D) conjunctivitis.

C Herpes zoster ophthalmicus is an emergent condition involving a shingles infection to the ophthalmic division of the trigeminal nerve. Hutchinson sign (involvement of the nose) is a likely sign in the earlier stages of infection that the eye will be involved. Complications from this condition include postherpetic neuralgia and vision loss. Conjunctivitis usually presents with redness and swelling to the conjunctiva and drainage, none of which is reported or observed with this patient's assessment. Unilateral vision changes could be a sign of an ischemic stroke; however, fever, rash and fatigue make this less likely in this patient. Assessment of intact EOM helps to rule out orbital cellulitis.

The nurse is assessing a patient with a small pinhole-sized wound to the fingertip from an injection puncture while cleaning a paint gun. The nurse recognizes the highest priority of care is? A) application of topical antibiotic. B) wound cleansing with soap and water. C) surgical consult for wound debridement. D) placement of a steri-strip.

C High-pressure puncture wounds, even as small as a pinhole to the fingertip, caused by paint or grease guns are a serious, limb-threatening emergency. The material (paint/grease) is injected into the soft tissue of the involved limb causing severe tissue damage and swelling. These injuries are high risk for infection with high morbidity and mortality rates due to delayed identification and deep tissue necrosis. The highest priority is surgical wound debridement under general anesthesia, drain placement and sterile dressing application. Wound cleansing with soap and water, steri-strip placement and application of topical antibiotics are not the highest priority for this patient as they do not treat the severity of the high-pressure injury.

A patient is being treated in the ED for a heat stroke while working outdoors replacing roofing shingles. The nurse anticipates which of the following findings? A) hypertension and bradycardia B) hypotension and bradycardia C) hypotension and tachycardia D) hypertension and tachycardia

C Hypotension, tachycardia, tachypnea, and altered mental status as well as an elevated core temp (greater than 104F rectal) are anticipated findings in a patient experiencing a heat stroke following excessive outdoor temperatures and humidity. Hypertension and bradycardia are not expected findings for a patient experiencing heat stroke.

A patient who sustained a leg laceration from a chainsaw is unsure if he has ever received a tetanus immunization. The patient should receive? A) 5 mL of immune (gamma) globulin. B) 0.5 mL of tetanus, diphtheria toxoids. C) 250 U of tetanus immune globulin and 0.5 mL of tetanus, diphtheria toxoids, and pertussis vaccine. D) 0.5 mL of tetanus, diphtheria toxoids, and pertussis vaccine.

C If the patient has not received an initial tetanus series or has only received one of the series, the patient should be started on a regimen of 0.5 mL of dT. If the patient has a grossly contaminated wound, simultaneous administration of 250 units of intramuscular antitoxin is recommended.

Orthopnea, diaphoresis, anxiety, moist crackles (rales), and cough are usually associated with? A) pulmonary embolism. B) bronchial asthma. C) pulmonary edema D) pneumonia.

C In left ventricular failure, the cardiac output declines and pulmonary venous pressure increases. This results in an accumulation of fluid in the pulmonary tissue and alveoli.

A patient presents with pain and swelling to the hand after being hit with the stream of a high-pressure grease gun. It would be a PRIORITY for the nurse to? A) apply a sterile dressing to the hand. B) apply direct pressure to the hand. C) prepare patient for the operating room. D) prepare the patient for a MRI.

C Injuries from high-pressure guns are serious and immediate surgical intervention is required to drain the paint or oil and to preserve tissue.

Upon initial assessment of a male patient involved in a motor vehicle crash, the nurse observes bruises in various stages of healing to his torso. He denies any medical history. When asked about the bruising, the patient avoids eye contact, states that he fell yesterday, and asks if his male partner has arrived to the ED. The nurse suspects? A) the bruising is a result of the stated fall. B) the bruising was sustained during the motor vehicle crash. C) the patient may be a victim of domestic violence. D) the patient may have undiagnosed coagulopathy.

C Interpersonal violence is leading cause of injury for gay men. The coloration of the bruises does not correlate with the time of the MVC or stated falls.

A young adult with a history of General Anxiety Disorder presents to the ED and appears to be extremely anxious and diaphoretic. They state they feel like their "heart is beating out of their chest." The ED nurse understands the INITIAL course of treatment for this patient is to? A)perform a mental health screening B) administer anxiolytic medication C) perform an ECG D) administer antipsychotic medication

C Life-threatening emergencies should always be ruled out, even if the symptoms are likely contributed to a mental health emergency and not a physiological condition. Organic illness, medications, drug abuse, and obvious psychotic causes of an anxious state must be ruled out and documented before treatment of anxiety.

A neonate has just been delivered in the emergency department. After drying, warming, and suctioning, additional intervention is necessary if the neonate has? A) a skin temperature of 98°F (36.7°C). B) respirations of 40 breaths/min. C) a heart rate of 80 beats/min. D) dusky hands.

C Neonatal Resuscitation standards state that if the infant's heart rate is less than 100 beats/min, respiratory interventions are necessary; if less than 60, then cardiac compressions must be started. Dusky extremities (acrocyanosis) are normal for the neonatal period. Respirations of 40 to 60 breaths/min is normal for the neonate. Skin temperature of 98°F (36.7°C) is normal for the neonate.

Which of the following indicates significant dehydration in a 6-month-old infant with a history of vomiting and diarrhea? A) cries loudly during the physical examination B) has three loose stools in 3 hours C) does not urinate for more than 8 hours D) has two bouts of projectile vomiting

C Normal urinary output in a well-hydrated child is 1 mL/kg/h. Decreased urine output is a sign for the need to be evaluated in an emergency department setting.

A patient presents after being splashed in the eyes with a cleaning solution. Corneal clouding is noted bilaterally and an order is given to irrigate the patient's eyes. Irrigation may be discontinued after? A) Each eye has been irrigated with 1,000 mL of irrigation solution. B) The pain is a 2 on a 0-10 scale. C) The ocular pH reaches 7.0 bilaterally. D) 2 hours have passed.

C Ocular pH ranges from 7.0 - 7.3. Irrigation must be maintained until pH is within normal range. There is no minimun time frame, fluid irrigant, or pain scale rating recommended as obtaining a normalized ocular pH is the goal.

Assessment of a patient who has acute cholecystitis is MOST likely to reveal? A) pain in the left upper quadrant of the abdomen and positive results on a guaiac test of vomitus. B) pain that radiates to the left shoulder and positive results on a stool guaiac test. C) pain in the right upper quadrant of the abdomen and slight scleral icterus. D) epigastric pain that radiates to the back and watery stools.

C Pain in the right upper quadrant of the abdomen is caused by inflammation of the gallbladder and obstruction of the cystic duct from a gallstone. The icterus is caused by excess circulating bilirubin secondary to backup from common duct obstruction.

A child with an extremity fracture is being prepared for admission prior to surgery. The child has a low-grade fever and may have been exposed to chickenpox. The nurse should? A) advise the receiving nurse to initiate isolation if vesicles develop. B) anticipate administering diphenhydramine (Benadryl). C) isolate the patient in a negative-pressure room. D) anticipate administering methylprednisolone (SoluMedrol).

C Patients are placed in isolation in a negative-pressure room because chickenpox is a highly contagious disease caused by the varicella zoster virus. The virus is spread by direct contact or droplets, or aerosolized from skin lesion fluid or respiratory tract secretions. Patients experience a 14- to 16-day incubation period and are contagious for 1 to 2 days before the rash begins and are no longer contagious when the rash is crusted over.

An adult patient presents to the ED after suffering an electrical injury to 25% total body surface area. The goal for fluid resuscitation is? A) 2ml x TBSA (%) x body weight (kg) B) urine output 30 ml/hr C) urine output over 100 ml/hr D) 6ml x TBSA (%) x body weight (kg)

C Patients suffering from electrical burns require higher levels of fluid resuscitation due to the risk of hemoglobinuria and myoglobinuria. The goals of resuscitation are reflected by urine flow, not volume given. Urine output goals for the adult population are 100 ml/hr. 30 ml/hr is not enough for an adult patient. The formulas provided are not utilized for fluid resuscitation for electrical burns.

A patient in the emergency department has just been diagnosed with an orbital fracture. The nurse instructs the patient to? A) Apply a warm compress to the eye B) Lay supine C) Avoid blowing the nose D) Apply direct pressure to the eye

C Patients that have an orbital fracture should avoid blowing their nose, performing the Valsalva maneuver or other activities that may force air from the sinuses into subcutaneous tissue around the fracture. Direct pressure should not be applied in order to prevent further injury to the globe. A cold compress should be applied, and the head of bed elevated, to decrease swelling.

A pediatric patient presents to the emergency department with vomiting, diarrhea, abdominal pain, lethargy and tachycardia that started after eating. The parents report no one else in the family ate the same food. The nurse should anticipate a stool sample for? A) C. Difficile. B) helicobacter pylori. C) ova and parasite. D) occult blood.

C Patients with gastroenteritis should have stool samples taken for ova and parasites. While diarrhea is a symptom of C. Difficile, it is usually associated with antibiotic use. There is nothing to indicate that the patient may have bleeding in the stool. Helicobacter pylori is tested in cases of chronic gastric or duodenal ulcers and dyspepsia.

Assessment of a patient with severe acute pancreatitis reveals ascites and diminished breath sounds in the right lower lobe. Vital signs are: BP 109/70, HR 110 beats/min, RR 38 breaths/min The nurse should suspect that the patient has developed? A) pneumonia. B) a pneumothorax. C) pleural effusion. D) a diaphragmatic hernia.

C Pleural effusion is an excess of fluid trapped in the intrapleural space of the lungs. In pancreatitis, this occurs because fluid associated with ascites moves from the peritoneal space into the pleural space through lymph channels. Seventy percent of pleural effusions occur on the right side.

The nurse knows that the discharge education for a patient with esophagitis is successful when the patient states? A) "I will lay flat at night." B) "I will eat three large meals a day." C) "I will eat smaller meals throughout the day." D) "I will take antibiotics as directed."

C To minimize the symptoms of esophagitis, the patient should elevate the head of the bed and avoid large volumes of food or drink. Proton pump inhibitors or histamine blockers may be prescribed, but antibiotics are not indicated for the treatment of esophagitis.

A patient presents to the emergency department with increasing back pain, new onset incontinence, and bilateral lower extremity weakness. The patient has a history of COPD, diabetes, hypertension, hyperlipidemia, and prostate cancer. The nurse understands the MOST likely cause of the patient's symptoms is? A) abdominal aortic aneurysm. B) pulmonary embolism. C) metastatic spinal cord compression. D) subacute ischemic stroke.

C Prostate cancer has a relatively high incidence of metastases to the spinal column. The presenting signs and symptoms point to likely metastatic spinal cord compression. The other diagnoses are less likely to be present with these symptoms.

The nurse is assessing a patient who presents with multiple injuries from a fall, including bruises in various stages of healing that are not consistent with the history provided. The family member accompanying the patient insists on answering for the patient even though the patient is awake, alert and able to answer questions on their own. The nurse should be MOST concerned about? A) depression B) dementia C) abuse D) neglect

C Red flags for abuse include an inconsistent history of injury with injuries that are not logically caused by the stated history and an overbearing individual who speaks for the patient. Neglect is a failure of the caregiver to provide the necessities of life. There is no indication of depression and dementia as the patient is awake and alert and able to answer questions.

The emergency nurse is triaging a patient with a suspected serotonin syndrome, the nurse knows that major signs and symptoms of this syndrome are agitation, diarrhea, seizures, and ? A) visual disturbances. B) paralysis. C) hyperthermia. D) dry mouth.

C Serotonin syndrome occurs when high levels of serotonin build up in the body. This can result from patients that take medications such as antidepressants and mood stabilizers. Agitation, diarrhea, seizures and hyperthermia are signs and symptoms of serotonin syndrome.

The nurse is caring for a patient who presents with severe posterior epistaxis. The PRIORITY intervention is to? A) obtain nasal packing supplies. B) assess for anticoagulant use. C) suction the airway D) assess blood pressure.

C Severe posterior epistaxis is frequently the result of arterial bleeding and the patient is often vomiting/coughing up a significant amounts of blood. Suction equipment is what will assist in maintaining the patency of the airway and therefore should be the initial priority. Assessing blood pressure/anticoagulant use and obtaining nasal packing are indicated, but are not the highest priority.

An infant in shock will MOST likely be? A) difficult to arouse, but will respond to its parents. B) irritable and hyperreflexive. C) lethargic and have mottled skin and poor capillary filling.Comments: D) active and demonstrate tachypnea.

C Shock is inadequate tissue perfusion. As vital organ perfusion decreases, compensatory mechanisms shunt blood to the core resulting in cool, mottled skin and decreased peripheral pulses. Decreased perfusion to the brain may initially cause restlessness, but will ultimately cause lethargy as the shock state overwhelms the infant's ability to compensate.

A patient presents to the emergency room post traumatic all-terrain vehicle accident in a forest. During the initial assessment, several wounds are noted on the patient's limbs with an appearance suspicious for foreign bodies. What imaging test would the nurse anticipate being ordered for detection of foreign bodies? A) X-Ray B) Ultrasound C ) Computed tomography (CT) D) Magnetic resonance imaging (MRI)

C Since this accident was in a forest, the most common types of foreign bodies would be wood. CTs are more useful in this. MRIs can be used if foreign bodies are suspected and are not found by other means. X-Rays are useful in finding glass, and ultrasounds would be most useful if identifying a foreign body in the posterior portion of the eye.

The triage nurse should prioritize a patient who has ? A) hypertension and a headache. B) a cough and fever. C) stridor after being stung by a bee. D) abdominal pain and distention.

C Stridor is indicative of an upper airway obstruction. After being stung by a bee, stridor should cause high suspicion for anaphylaxis and requires immediate intervention to protect the patency of the airway. While the other conditions are concerning, stridor can be a sign of airway compromise and should be the priority.

After a non-traumatic resuscitation, the patient has a perfusing ECG rhythm and a stable blood pressure, but the patient remains comatose. The nurse should NEXT prepare for? A) transport to the cardiac catheterization lab. B) transport to the neurovascular suite. C) initiation of targeted temperature management. D) initiation of stroke protocol.

C Targeted temperature management should be implemented for patients who remain comatose after return of spontaneous circulation (ROSC). There is no obvious sign of stroke and no indication to send the patient to the cardiac catheterization lab or the neurovascular suite.

The treatment of choice for a patient presenting with headache in the right temporal region and jaw pain is? A) morphine sulfate, IV. B ) administration of oxygen. C) dexamethasone (Decadron), IV. D) sumatriptan (Imitrex), SQ.

C Temporal arteritis is an inflammatory condition, so a steroid is the first-line treatment to prevent vision loss.

A patient with complaints of jaw pain and muscle spasms to the left side of their face reports increased emotional stress and teeth grinding while sleeping. The nurse suspects? A) Bell palsy B) trigeminal neuralgia C) temporomandibular joint dislocation D) dental caries

C Temporomandibular joint dislocation is associated with unilateral or bilateral jaw pain and muscle spasms due to the displacement of the mandibular condyle from its position of function. Risk factors include poor fitting dentures, malocclusion, teeth grinding or clenching and emotional stress which may alter muscular balance. Bell palsy would cause unilateral facial droop, however not the cause of the patient's symptoms. Trigeminal neuralgia would have symptoms of unilateral facial pain, however these pains would be recurrent, severe and distributed along the branches of the 5th cranial nerve, not associated with emotional stress or bruxism. Dental caries can cause jaw and facial pain, however they are not the most likely cause of this patient's condition.

The triage nurse has several patients with psychiatric complaints. The nurse understands that the HIGHEST priority is the patient with? A) suicidal ideation accompanied by a parent. B) restlessness and confusion. C) violent behavior and disheveled appearance. D) paranoid delusions & auditory hallucinations.

C The most critical need is the patient with violent behavior and diminished self-care capacity. "The goal of mental health triage is to determine the severity of the presenting problem while protecting the patient, staff, and others. Individuals with mental health emergencies commonly exhibit intense personal distress, suicidal ideation, and self-neglect that jeopardizes their health and safety." Violent behavior demonstrates high levels of distress and is the most immediate threat to the safety of the patient and others.

An unresponsive patient arrives to the ED after falling and hitting their head. Family reports that the patient initially lost consciousness but was alert and answering questions while en route to the ED. The nurse suspects? A) subarachnoid hemorrhage B) subdural hematoma C) epidural hematoma D) cerebral contusion

C The nurse knows that there is often a pattern of unconsciousness following an epidural hematoma where the patient is initially unconscious followed by a lucid interval then unconsciousness again. A subarachnoid hemorrhage is usually characterized by a sudden onset severe headache. While loss of consciousness is possible with a subdural hematoma and cerebral contusion, they do not follow the pattern like an epidural hematoma.

The nurse is caring for a patient that was involved in a house fire and has burns to their chest and arms. The nurse initiates this action in order to stop the burning process? A) Initiate fluid replacement B) Administer pain medication C) Remove all clothing D) Apply cool dressings

C The nurse knows to stop the burning process all clothing and jewelry must be removed. These items can hold in heat against the patient's skin. Administering pain medications, applying dressings and initiating fluid replacement are part of a burn patient's care but will not stop the burning process.

A patient with a history of heart failure presents with shortness of breath and is unable to speak in complete sentences. A chest x-ray shows a large collection of fluid in the pleural space. The PRIORITY intervention is? A) administration of antibiotics. B) a paracentesis. C) a thoracentesis. D) insertion of urinary catheter.

C The patient has a pleural effusion. Because the pleural effusion is large, and the patient is in respiratory distress, a thoracentesis is indicated to remove the fluid and improve the patient's respiratory status. Paracentesis is a procedure to remove fluid from the peritoneal cavity, not from the pleural space. The patient may require antibiotics if there are signs of infection, but this is not the priority intervention based on the x-ray findings. Due to the risk of infection, a urinary catheter should only be inserted if indicated.

The nurse is caring for a patient who is unable to stand and complaining of lower back pain and urinary retention. The nurse understands which intervention provides definitive diagnostic information for their condition? A) lumbar puncture B) CT of lumbar spine C) MRI of lumbar spine D) x-ray of lumbar spine

C The patient has signs of Cauda Equina Syndrome and requires an emergency MRI. A CT or x-ray of the lumbar spine will not provide the details needed to guide a diagnosis. A lumbar puncture is not indicated.

A patient has arrived to the emergency department after diving into a shallow pool. The CT scan shows a spinal cord injury at the level of C3. The patient's current blood pressure is 73/42 mmHg and heart rate is 62 beats per minute. The nurse prepares to administer which of the following? A) additional fluid bolus B) blood products C) norepinephrine bitartrate (Levophed) D) phenylephrine (Neo-synephrine)

C The patient is demonstrating signs of neurogenic shock. Norephinephrine bitartrate is used in the treatment of neurogenic shock for its vasoconstrictive properties. Phenylephrine should be avoided in patients with cervical and high thoracic spinal injuries because it can cause increased bradycardia. Blood products and additional fluid administration is used for volume expansion, which is typically not required in a patient with spinal shock.

The nurse is caring for a patient who is 2 days postpartum, is hypertensive and complaining of a severe headache. The nurse should prepare to administer? A) oxytocin (Pitocin). B)methylergonovine maleate (Methergine). C) magnesium sulfate. D) acetaminophen.

C The patient is experiencing symptoms of preeclampsia, the primary treatment for which is magnesium sulfate, along with blood pressure management.

A patient who returns to the emergency department 3 days after a motor vehicle crash has pain in the left upper quadrant of the abdomen. Assessment reveals pain on palpation, and the patient is hemodynamically stable. The nurse should be MOST concerned about the possibility of? A) a costochondritis. B) a pneumothorax. C) a splenic hematoma. D) bleeding from a liver laceration.

C The spleen, located in the left upper abdominal quadrant, is the most commonly injured organ in patients with blunt abdominal trauma. Because the spleen (like the liver and kidneys) is an encapsulated organ, bleeding from the parenchyma is commonly well-contained within the capsule. Therefore, the patient remains hemodynamically stable and there is no free blood in the peritoneum. Most patients with a splenic subcapsular hematoma will heal nicely without intervention. However, the capsule can rupture, releasing blood into the abdominal cavity. A pneumothorax is manifested by respiratory findings, costochondritis is characterized by pain with inspiration or chest palpation, and a liver laceration causes pain in the RIGHT upper abdominal quadrant.

A patient has an open pneumothorax secondary to a gunshot injury. What is the proper sequence of steps in management? A) Initiate an IV of lactated Ringer's solution, check for a pulse, and place the patient on the affected side. B) Obtain baseline vital signs, initiate an IV of lactated Ringer's solution, and place the patient on the unaffected side. C) Apply an occlusive dressing to the wound, check for a pulse, and initiate an IV of normal saline solution. D) Initiate an IV of normal saline solution, apply an occlusive dressing to the wound, and prepare for intubation.

C The steps taken in the key follow the principles of the Trauma Nursing Process. In the Primary assessment, all elements assessed are of such a critical nature that any major deviations from normal require immediate intervention. This answer addresses the "C" or "circulation" of the primary survey. A life-threatening wound has been identified. Upon identification, the nurse must apply an occlusive dressing, check for a pulse, and initiate an IV of normal saline. These are all interventions under circulation and follows the Primary assessment and Intervention rules for trauma.

A patient is being discharged home with a diagnosis of esophagitis related to gastric esophageal reflux disease (GERD). Which of the following statements indicates the patient understands the discharge instructions? A) "I will drink orange juice whenever I am having GERD symptoms." B) "I will gain weight to help improve my condition." C) "I will refrain from laying down after eating." D) "I will take my antibiotics until I feel better."

C To prevent more irritation to the esophagus, sitting up after eating will decrease the stomach acid rising into the esophagus. Citrus fruits are considered acidic and can irritate the esophagus. A course of antibiotics should be taken until they are completed, not until the patient is feeling better. Weight loss, not gain, has been proven to help with GERD-induced esophagitis.

A conscious patient arrives via EMS following a motor vehicle crash with a mangled lower extremity. What is the nurse's priority assessment? A) Airway assessment B) Pedal pulse evaluation C) Uncontrolled hemorrhage D) C-spine stabilization

C Uncontrolled hemorrhage is the major cause of preventable death after injury. The primary across the room assessment is the priority assessment for a trauma patient, it includes uncontrolled hemorrhage and alertness. Maintaining c-spine stabilization, assessing the patient's airway, and assessing pulses are all important but do not take priority over the need to stop uncontrolled hemorrhaging.

The triage nurse is called to assist a laboring patient in their vehicle. The significant other states that the baby's head delivered just before arrival, but the shoulders seem to be preventing complete delivery. The nurse recognizes which of the following as the PRIORITY intervention for the neonate? A) perform the initial APGAR score B) gently pull light pressure on the posterior shoulder C) suction the mouth, then the nose D) clamp the umbilical cord 4-5 cm from the umbilicus

C When a neonate delivers quickly, the nurse should prioritize drying and warming the infant and then suctioning if needed. For a baby that requires an assisted and longer delivery, the baby's mouth and then nose may be suctioned to prevent aspiration of amniotic fluid and blood. Clamping the cord should not occur until the baby is delivered OR if the cord is pulled tightly and the practitioner feels it is better to clamp and cut the cord after delivery of the head than to risk other complications. The APGAR score should be performed after full delivery of the neonate. If assisting with delivery of the shoulders, the anterior shoulder should be delivered first, followed by the posterior shoulder.

An indwelling urinary catheter is ordered for a patient who has sustained a straddle injury to his external genitalia. Before catheter insertion, the nurse should assess the patient for? A) penile tenderness. B) scrotal or penile swelling. C) blood at the urinary meatus. D) the ability to void spontaneously.

C When assessing a trauma patient with blood at the urinary meatus, it is important to know that this is a sign of a possible pelvic fracture and that a urinary drainage catheter insertion is contraindicated. Penile tenderness and scrotal or penile swelling are expected complaints after the described trauma, but alone they are not a contraindication for a urinary drainage catheter insertion. The ability to void spontaneously is not an indication or contraindication for the insertion of a urinary drainage catheter.

Which of the following pieces of information is useful in the assessment of a patient with suspected hepatitis A? A) unusual odor of feces B)history of diarrhea C) color of stools D) time of last bowel movement

C With hepatitis A, stool becomes clay-colored due to loss of pigmentation.

A patient who is trembling, sweating, and hyperventilating is having an anxiety attack. Arterial blood gas analysis is likely to show which of the following? A) oxygen saturation < 96% B)pH < 7.35 C) PaCO2 < 35 mm Hg D) HCO3-level > 26 mEq/L

C With the increased respiratory rate that occurs with anxiety, carbon dioxide is blown off, which results in a low PaCO2.

The nurse is caring for a patient who sustained severe crush injuries after being trapped under a car. Upon arrival to the trauma bay, the patient requires intubation. The nurse should seek clarification with which of the following orders? A) ketamine (Ketalar) B) etomidate (Amidate) C) succinylcholine (Quelicin) D) rocuronium (Zemuron)

C The treatment team should be cautious of hyperkalemia with crush injuries as the damaged tissues and cells release potassium into the system. Succinylcholine is known to increase hyperkalemia and in the presence of existing hyperkalemia can result in dysrhythmias. Depending on the timeframe in which the original injury occurred, succinylcholine may still be safe but it would be prudent of the nurse to clarify this with the provider prior to administration.

A patient who sustained a gunshot wound is bleeding from the arm. After applying a pressure dressing, the nurse should next evaluate? A) presence of bleeding at the dressing. B) presence of pain. C) development of fever. D) changes in neurovascular status.

D A gunshot wound is a missile injury. Bullet wounds may cause neurovascular, bony, and soft-tissue injuries remote from the projectile path. There is a potential for occult neurovascular injury, especially in high-velocity injuries to the arm. After the intervention of applying a pressure dressing, signs of circulation should be reassessed. Alterations in the neurovascular status indicate further complications. Fever would generally not be seen immediately following an injury upon arrival in the emergency department. Unless there is indication of hemorrhage, a normal amount of bleeding is expected following the initial application of a pressure dressing. Reassessment of any interventions associated with airway, breathing, circulation, and treating underlying conditions related to disease or injury determines the priorities of care.

The nurse has been caring for a patient with COPD over the last several hours when they start becoming slightly confused and drowsy. Vital signs are:BP 130/85 mmHgHR 95 beats/minRR 28 breaths/minSpO2 90% (2 liters nasal cannula) The nurse should request an order for? A) high flow nasal cannula. B) ammonia level. C) MRI of head. D) arterial blood gas.

D A patient with COPD may experience confusion and changes in level of consciousness with hypercapnia. An arterial blood gas should be ordered to determine whether the patient requires any noninvasive positive pressure ventilation. Ammonia level and MRI of the head are not indicated based off the information provided. High flow nasal cannula would not be appropriate as the patient's current SpO2 is appropriate for a patient with COPD.

A patient suspected of having anorexia nervosa is being triaged. The emergency nurse expects which blood abnormality commonly related to this disease? A) increased glomerular filtration rate B) decreased blood urea nitrogen C) hyperkalemia D) metabolic acidosis

D A patient with anorexia nervosa will have decreased glomerular filtration rate, elevated blood urea nitrogen, hypokalemia and metabolic acidosis. These laboratory changes are due to excessive vomiting and dehydration.

An alert and oriented older adult is diagnosed with a STEMI. Despite being informed of the risks, the patient still refuses treatment. The patient's medical power of attorney is demanding that the patient be treated anyway. The nurse understands the right for medical decision making lies with? A) the ethics committee. B) the physician. C) the medical power of attorney. D) the patient.

D A patient's wishes can only be overridden when the patient has been deemed incapable of making their own decisions.

After a patient has an episode of violent behavior in the waiting room, an important intervention is to? A) tell the patient to get himself under control. B) allow the patient to go home as soon as possible. C) instruct the patient to be quiet and speak only when spoken to. D) encourage the patient to verbalize his feelings about the situation.

D A primary intervention for patients with violent behavior is to "encourage the patient to identify feelings."

An adult patient sustains blunt trauma to the abdomen in a motor vehicle crash. The patient refuses to allow any invasive treatments. Initial assessment reveals an ecchymotic area in the left upper quadrant. The nurse should? A) request an order to sedate the patient. B) allow the patient to leave after signing a medical release form. C) obtain consent from a relative for treatment. D) assess the patient's neurological status

D A rational, competent adult can refuse any treatment. The first step is to establish the patient's ability to understand the consequences of refusing treatment. Due to the mechanism of injury, the patient should be evaluated for the presence of a head injury.

Which of the following triage assessments in a toddler indicates the need for IMMEDIATE treatment? A) systolic blood pressure of 90 mm Hg B) temperature of 102°F (38.8°C) C) sinus arrhythmia of 140 beats/min D) respirations of 60 breaths/min

D A respiratory rate of 60 breaths/min is a triage red flag. Children have a decreased pulmonary reserve. A child breathing this fast for any extended period of time without intervention will decompensate very quickly into respiratory failure.

A patient in the emergency department is receiving a 1 g IV infusion of phenytoin (Dilantin) for control of seizures. During the infusion, the patient's QRS complex widens markedly. The nurse should turn off the infusion and prepare for possible? A) hypertensive crisis. B) extravasation at the IV site. C) status epilepticus. D) cardiac arrest

D A side effect of phenytoin (Dilantin) is cardiac collapse and hypotension. If symptoms develop, such as a widened QRS, the infusion should be discontinued immediately. The patient should be monitored closely thereafter.

Which of the following statements about subdural hemorrhage is true? A) It occurs between the skull and the dura. B) It results from a tear in the middle meningeal artery. C) It results from a thrombosed artery. D) It may initiate an acute or chronic onset of symptoms.

D A subdural hemorrhage is usually due to venous bleeding that results from tearing of bridging veins. An acute subdural hemorrhage displays signs and symptoms within 48 hours after injury. A chronic subdural hemorrhage may not display signs or symptoms for 2 weeks or longer.

A patient presents to the emergency department with sudden onset severe right eye pain, nausea and vomiting. The right eye is fixed and dilated with injected sclera. There is a history of hyperopia corrected with contact lenses. The nurse understands this patient is likely experiencing? A) conjunctivitis. B) epithelial keratitis. C) ophthalmic herpes zoster. D) acute angle closure glaucoma.

D Acute angle closure glaucoma often presents as a rapid, progressive condition affecting one or both eyes through visual impairment. This condition causes severe ocular and periocular pain and patients often have associated nausea and vomiting. On assessment, the pupil is often fixed and semi-dilated, ocular redness and corneal edema are present, and intraocular pressure is elevated.

A patient would MOST likely require rabies prophylaxis if they? A) handled raccoon stool. B) came in contact with a person diagnosed with rabies. C) were bitten by a neighbor's dog. D) were bitten by a bat.

D Bats commonly carry rabies, so prophylaxis treatment is recommended if bitten. Contact with a person with rabies does not cause transmission of the disease. While raccoon's may carry the rabies disease, it is not transmitted in their stool. A bite from a neighborhood dog may not require initial rabies prophylaxis as the dog can be quarantined.

When caring for a patient with a suspected blunt cardiac injury, the nurse should FIRST prepare to administer? A) sublingual nitroglycerin. B) a narcotic analgesic. C) lidocaine (Xylocaine). D) supplemental oxygen.

D Blunt cardiac injury, formerly referred to as cardiac contusion or myocardial contusion, is best diagnosed with an echocardiogram and is treated symptomatically. The patient's cardiac rhythm should be monitored, supplemental oxygen should be administered, and the patient's musculoskeletal chest pain should be treated. Dysrhythmias (premature ventricular complexes) are common, but are rarely treated unless they interfere with hemodynamic stability.

Nitroglycerin reduces myocardial oxygen consumption because its primary effect is to? A) enhance contractility. B) increase afterload. C) slow the heart rate. D) reduce preload.

D By promoting vasodilation, nitroglycerin reduces blood return to the heart. The reduction in preload means that the heart does not have to work so hard. When the heart's workload is decreased, myocardial oxygen consumption drops and cardiac output usually improves.

Nitroglycerin reduces myocardial oxygen consumption because its primary effect is to? enhance contractility. slow the heart rate. increase afterload. reduce preload.

D By promoting vasodilation, nitroglycerin reduces blood return to the heart. The reduction in preload means that the heart does not have to work so hard. When the heart's workload is decreased, myocardial oxygen consumption drops and cardiac output usually improves.

The emergency nurse knows that a patient who has sustained a significant crush injury is at risk of which electrolyte abnormality? A) Hypercalcemia B) Hypocalcemia C) Hypokalemia D) Hyperkalemia

D Cellular death of the tissues involved in a significant crush injury, causes hyperkalemia and acidosis. Untreated hyperkalemia can lead to cardiac dysrhythmias and death. Potassium increases in crush injuries due to cellular necrosis.

The nurse notices that a co-worker has been chronically tardy, fatigued, apathetic, and increasingly distracted at work. The nurse suspects? A) a hangover. B) paranoia. C) drug diversion. D) compassion fatigue.

D Chronic tardiness, fatigue, apathy, and the inability to focus at work are some of the symptoms associated with compassion fatigue and should be recognized as such and the affected nurse be given the appropriate support and resources needed. A hangover, drug diversion, and paranoia do not chronically exhibit these symptoms specifically in the setting of the workplace.

The nurse is caring for a patient with multiple superficial abrasions. To prevent complications, the nurse anticipates? A) soaking the wounds in a betadine solution. B) providing pain control. C) administering antibiotics. D) cleansing the wounds.

D Cleansing is a critical intervention in the care of abrasions to prevent any complications such as infection. Pain control is important but will not prevent complications. Antibiotics may be needed only if a complication occurs and soaking the wounds in a betadine solution may cause further tissue damage.

Which of the following medications should a nurse anticipate administering initially to an adult patient in status epilepticus? A) flumazenil (Romazicon) B) phenobarbital C) fosphenytoin (Cerebyx) D) diazepam (Valium)

D Diazepam (Valium) is a benzodiazepine used as a first-line drug in the treatment of status epilepticus. Fosphenytoin (Cerebyx) is the second-line drug for seizure treatment after a benzodiazepine. Flumazenil (Romazicon) is a reversal agent for benzodiazepine overdose and can precipitate seizures. Phenobarbital is a long-term treatment to prevent seizure recurrence and is not a first-line medication for status epilepticus.

A trauma patient has received multiple transfusions in the emergency department and starts to ooze blood from the IV sites. This suggests the development of? A) sepsis. B) multisystem organ failure. C) hemophilia. D) disseminated intravascular coagulation.

D Disseminated intravascular coagulation (DIC) involves simultaneous clotting and bleeding and is associated with various diagnoses, including trauma and multiple transfusions of banked blood. The patient may have acute bleeding or gradual blood loss. DIC is characterized by a prolonged partial thromboplastin time (PTT) and oozing from a wound or puncture sites. Blood administration is not without risk, as it can contribute to depletion of clotting factors that contribute to DIC.

A patient is being treated for increased intracranial pressure related to a traumatic brain injury with mannitol (Osmitrol). What should the emergency nurse be aware of when administering this medication? A)Tinnitus B) Oliguria C) Glycosuria D) Hypokalemia

D Due to the diuretic effect of mannitol (Osmitrol), hypokalemia can occur. Diuresis, not oliguria, is an expected effect of this medication. Tinnitus and glycosuria, glucose in the urine, are considerations when administering Lasix (furosemide).

A patient diagnosed with pneumonia should have the sepsis protocol initiated when which of the following vital signs are noted? A) T (oral) 97.2°F (36.2°C), HR 125 beats/min, white blood cell count 8,000/mm3 B) T (oral) 99.4°F (37.4°C), HR 98 beats/min, white blood cell count 11,000/mm3 C) T (oral) 100°F (37.8°C), HR 85 beats/min, white blood cell count 6,000/mm3 D) T (oral) 101.5°F (38.6°C), HR 115 beats/min, white blood cell count 14,000/mm3

D Early recognition of sepsis is key to preventing death. If a patient has a new suspicious/confirmed infection and if two of the following signs or symptoms are present, sepsis protocols should be initiated. T > 100.8°F or < 96.8°F HR > 90 beats per minute RR > 20 breaths per minute Acutely altered mental status WBC > 12,000 mm or < 4,000 mm Serum glucose > 120 mg/dL in the absence of diabetes.

The earliest indication of increased intracranial pressure is? A) abnormal respirations. B) widened pulse pressure. C) altered level of consciousness. D) decreased heart rate.

D Early signs and symptoms of increased intracranial pressure are changes in the level of consciousness and restlessness. Vital sign changes are a late sign of increased intracranial pressure.

The earliest indication of increased intracranial pressure is? A) widened pulse pressure. B) decreased heart rate. C) abnormal respirations. D) altered level of consciousness.

D Early signs and symptoms of increased intracranial pressure are changes in the level of consciousness and restlessness. Vital sign changes are a late sign of increased intracranial pressure.

An experienced nurse is now studying to pass a specialized nursing certification exam. When their co-workers question why they are doing this, the nurse responds that it is part of their? A) evidence-based practice. B) ethical practice. C) team collaboration. D) lifelong learning plan.

D Lifelong learning keeps the nurse up to date on ever changing treatments and understanding of disease processes. Team collaboration is working with other members of the healthcare team. Evidence based practice integrates evidence and research findings into the nurse's practice. Ethical practice involves the nurse using ethical principles of autonomy, beneficence, justice and non-maleficence.

A patient who is at 36 weeks gestation and has eclampsia is receiving a magnesium infusion. Signs and symptoms of magnesium toxicity include? hypertension. hallucinations. hyperreflexia. hypoxemia.

D Magnesium is a smooth muscle relaxer; therefore, the patient should be monitored for respiratory depression. Hyperreflexia, hallucinations, and hypertension are all signs of magnesium deficiency. Hypoxemia would be a sign of respiratory depression.

A patient presenting with headache, fever and a stiff neck has cloudy cerebrospinal fluid from the lumbar puncture. Which isolation precautions should the nurse implement? A) standard B) airborne C) high risk D) droplet

D Meningitis is transmitted via the respiratory route and therefore the correct isolation precautions is droplet.

The nurse is caring for a patient who has an ectopic pregnancy. The patient has stable vital signs and states that she has an appointment with her obstetrical physician tomorrow. The nurse should prepare the patient for? A) admission to the operating room. B) administration of Rh immune globulin (RhoGAM) C) a repeat ultrasound examination. D) administration of methotrexate (Folex)

D Methrotrexate (Folex) administration is optimal in a hemodynamically stable patient who can follow up with their physician. Admission to the operating room is not indicated since the patient is hemodynamically stable. The patient has already been diagnosed, so there is no indication for a repeat ultrasound. Rh immune globulin (RhoGAM) would only be indicated if the patient has vaginal bleeding and is Rh negative.

A patient presents with dizziness, fatigue, and chest pain for the past 2 hours and reports using an albuterol inhaler (Ventolin) more than usual. The patient's heart rate is 156 beats/min. The nurse anticipates? A) preparing for cardioversion. B) administration of diltiazem (Cardizem). C) preparing for defibrillation. D) administration of metoprolol (Lopressor).

D Metoprolol is a cardio-selective β-blocker used to manage tachycardia caused by an albuterol overdose due to albuterol's β-agonist effect. Diltiazem is a calcium channel blocker. Cardioversion is not recommended at this time due to the patient being hemodynamically stable. Defibrillation is not indicated in this situation.

The MOST beneficial action of morphine in the treatment of pulmonary edema is to? A) increase systemic vascular resistance. B) decrease cardiac reserve. C) increase oxygen consumption. D) decrease preload.

D Morphine sulfate has several beneficial effects in the patient experiencing pulmonary edema, particularly pulmonary edema due to heart failure. Morphine reduces pain and anxiety but, most importantly, by increasing venous capacitance (through vasodilation), less blood is delivered to the lungs.

A patient arrives to the emergency department with paranoid ideations, delusions, hallucinations and hypersensitivity to environmental stimuli. The nurse recognizes the initial priority is to? A) collect blood and urine samples for medical clearance. B) initiate a psychiatric hold from the physician. C) obtain a medical and psychological history from a family member. D) eliminate environmental risks for self and staff harm.

D Patients presenting in psych ED have a high risk for harming themselves and/or staff during their assessment. Patient and staff safety is the initial goal of treatment for this patient. The Joint Commission recommends eliminating risks by reducing supplies, ligature risk items and initiating one-on-one observation. Collecting blood and urine for medical clearance is an important assessment in this patient's care as the patient displays classic signs of cocaine psychosis however patient safety is always the highest priority. After obtaining a medical history and completing medical clearance, the physician may elect to initiate a psychiatric hold, however this is not the initial goal of treatment.

The nurse is preparing a patient for an irrigation procedure following chemical exposure to the eye. In order to prevent ventilatory complications due to supine positioning, the nurse should modify the procedure if the patient reports a history of? A) chronic back pain. B) vertigo. C) diverticulosis. D) congestive heart failure.

D Patients with a chronic history of congestive heart failure may need accommodations for positioning during eye irrigation. A supine position may not be tolerated. Vertigo, diverticulosis, and chronic back pain may require modification of positioning during eye irrigation for comfort, however, not due to respiratory or ventilation compromise.

A trauma patient is in the emergency department with suspected cardiac tamponade. The nurse should prepare to treat this condition with? A) needle thoracostomy. B) transthoracic pacing. C) anti-dysrhythmics. D) pericardiocentesis.

D Pericardiocentesis involves placing a drain (needle or catheter) through the chest wall and into the pericardial space in order to drain accumulated fluid. In a trauma patient, the involved fluid is blood. Blood can accumulate rapidly in the pericardial sac, a fairly nondistensible structure, causing the ventricles to both fill and empty poorly. This produces an acute decrease in cardiac output. Needle thoracostomy is indicated for the relief of tension pneumothorax (not a simple pneumothorax). Antidysrhythmic drugs treat dysrhythmias, and transthoracic pacing is for patients with inadequate conduction system activity.

To assess pronator drift, the nurse should instruct the patient to? A) slide the left heel of your foot down your right shin. B ) smile and show me your teeth. C) touch your finger to your nose, alternating your right and left hands. D) hold both arms straight out with your eyes closed.

D Pronator drift is an assessment parameter used to determine arm weakness potentially caused by stroke. Both arms are held out straight while the nurse assesses for drift or the patient's inability to hold one or both arms in place.

Which of the following substances should be used to counteract the effects of heparin? A) quinidine sulfate B) vitamin K C) sodium bicarbonate D) protamine sulfate

D Protamine sulfate neutralizes heparin. With the presence of protamine, partial thromboplastin that are prolonged because of a heparin effect are reduced.

A patient presents to the emergency department yelling and threatening someone that is not there. The patient appears to be agitated. After placing the patient in a private room the nurse's first action should be to? A) apply restraints. B) assess for injuries. C) assess for suicidal ideation. D) place the patient into a hospital gown

D Removing potential objects of harm from the patient promotes staff and patient safety. The patient may not be able to accurately answer screening questions for suicide at this time. While treating injuries is important, safety of staff is a priority. Restraints should be last resort when it comes to managing patients with psychiatric presentations.

A Spanish-speaking patient presents to triage indicating they are having pain in the right lower quadrant, and speaks enough English to answer basic questions. The RN's next action should be? A) Take the patient directly to a bed for examination B) Communicate with the English-speaking family member present for any clarification needed C) Ask a Spanish-speaking co-worker to complete the triage to ensure accuracy D) Access the services of a certified medical interpreter

D Research has shown that use of non-certified medical interpreters is more likely to lead to medical errors and omission of important medical information, which in turn, can lead to the patient not fully understanding procedures and treatment options. The use of relatives can violate patient privacy, and does not ensure accurate translation of medical information. While Spanish speaking coworkers would be a valuable resource, The Joint Commission requires hospitals to provide professional interpretation services to ensure quality and accuracy.

After an emergent delivery, the neonate is unresponsive. Stimulation, warming and suction are performed with no success. The immediate next step the nurse should take is to: A) administer weight-based epinephrine B) administer weight-based dextrose C) assess the 1-minute APGAR score D) assess the neonate's respiratory rate and heart rate

D Resuscitation begins with assessment of the neonate's heart rate and respiratory rate. A neonate with a heart rate of less than 60 beats per minute and/or decreased or absent respirations requires immediate lifesaving interventions such as intubation and chest compressions. These lifesaving measures should not be delayed to obtain 1- or 5-minute APGAR scores. Medications are usually last resort attempts during neonatal resuscitation efforts.

The nurse is caring for a patient who took an unknown amount of acetaminophen. The initial acetaminophen level was below the toxic level. The nurse should anticipate? transferring the patient to the critical care unit. monitoring vital signs for 2 hours. discharging the patient home. repeating the acetaminophen level in 4 hours.

D Serum acetaminophen levels of 200 mcg/mL or greater 4 hours after ingestion are considered toxic and treatment should be initiated. Acetaminophen levels may continue to rise up to 4 hours after ingestion of a toxic amount.

A patient with Guillain-Barré syndrome should be expected to have a tingling sensation in the extremities, ascension of symmetric paralysis, and A) a positive Babinski's sign. B) tonic-clonic seizures. C) spasticity. D) decreased deep tendon reflexes.

D Signs and symptoms of Guillain-Barré syndrome are tingling sensation in the extremities lasting for hours to weeks, severely decreased deep tendon reflexes, and a symmetric paralysis that begins in the lower extremities and ascends. This classic pattern is seen in 90% to 95% of patients.

An adolescent patient presents to the emergency department with complaints of sore throat, fatigue, malaise, and fever. The patient has cervical lymphadenopathy and pain across the upper abdomen. Which of the following is the BEST test to help make the diagnosis? A) posterior-anterior and lateral chest radiographs B) indirect laryngoscopy C) purified protein derivative (PPD) testing D) Monospot

D Signs and symptoms of infectious mononucleosis include fatigue, fever, sore throat, cervical lymphadenopathy, splenomegaly, and hepatomegaly. The Monospot is the diagnostic laboratory test used in the process of diagnosing mononucleosis. The symptoms described are classic to mononucleosis. The Monospot test is the best choice based on the symptoms described. An indirect laryngoscopy is not indicated. The symptoms are not an indication for a PPD test or chest radiographs.

A patient with a history of chronic pancreatitis presents with extreme abdominal pain and is requesting a pain medication refill. The nurse knows an alternative cause of the abdominal pain is MOST likely? A) alcohol withdrawal B) urinary tract infection C) renal stone D) opioid withdrawal

D Signs of opioid withdrawal can mimic symptoms of pancreatitis such as severe, constant epigastric pain that radiates to mid-back, and nausea/vomiting. Alcohol withdrawal doesn't typically cause abdominal pain. Discomfort from a kidney stone typically originates in the flank and radiates to the groin. Urinary tract infections usually present with suprapubic pain.

Which solution is BEST when covering an open orthopedic injury in moist dressings? A) hydrogen peroxide B) iodine-based solutions C) chlorhexidine D) normal saline

D Soft tissue absorbs some of the solution causing additional injury, therefore, normal saline is best.

A patient presents to the emergency department following a sexual assault that occurred approximately 6 hours ago, and reports that they were strangled with a belt until they passed out. The patient has bruising to their neck, and their voice is hoarse. The PRIORITY intervention the nurse should anticipate is ? A) allowing the patient to drink water to assess the ability to swallow. B) photographing of the bruises noted on the patient's neck. C) collecting the patient's clothing and placing it in a plastic bag. D) computed tomography angiogram (CTA).

D Suggested radiographic evaluation for suspected strangulation injury includes a head CT and CTA of the neck to assess for vascular damage. Photography would not take priority over a medical evaluation. A patient's clothing should be placed in a paper bag to prevent moisture collection. A patient should refrain from oral intake if forensic evidence collection is anticipated and evaluated by a physician for injuries.

A recently widowed 86-year-old male with multiple chronic health conditions is brought to the ED by a family member after they found multiple doses of the patient's medication missing. The patient is alert, oriented and able to participate in his initial assessment. The patient is at highest risk for? A) substance abuse. B) neglect. C) dementia. D) suicide.

D Suicide is a major public health problem especially within the elderly population. Non-Hispanic Caucasian men over the age of 85 years are the most likely to die of suicide with a rate of 49.8 per 100,000. Chronic illness and physical ailments also increase the likelihood of suicide attempt. The patient's mental status would allow him to participate in screenings for substance abuse and geriatric maltreatment as well as assist to rule out early onset dementia.

The nurse is caring for a patient who was stung by a jellyfish. The PRIORITY intervention is? A) Soak the affected area in sterile water B) Apply dry dressing to the affected body part C) Immobilize the affected body part D) Rinse the affected area with salt water

D The nurse should immediately rinse the wound with salt water because fresh water will stimulate the nematocysts that have not already fired. Applying a dry dressing and immobilizing the affected body part are not indicated in the treatment of a jellyfish sting.

A patient with a history of congestive heart failure arrives short of breath and unable to speak in complete sentences. A chest x-ray shows a large collection of fluid in the pleural space. The PRIORITY intervention for this patient is? A) a paracentesis B) administration of antibiotics C) insertion of urinary catheter D) a thoracentesis

D The patient has a large pleural effusion and is in respiratory distress, therefore a thoracentesis is indicated. This will remove the fluid and improve the patient's respiratory status. A paracentesis is the procedure to remove fluid from the peritoneal cavity not from the pleural space. The patient may require antibiotics only if there are signs of infection but this is not the priority intervention based on the x-ray findings. Due to risk of infection, a urinary catheter should only be inserted if indicated.

The nurse is caring for a patient who presents to the emergency department complaining of anxiety, chest pain and shortness of breath. The patient reports a recent decrease in mobility due to an orthopedic procedure. Which INITIAL diagnostic procedure has the highest priority? A) pulmonary arteriography B) echocardiogram C) ultrasound of the leg and pelvic veins D) CT scan of the chest

D The patient history and symptoms of anxiety, chest pain and shortness of breath are all indications of a pulmonary embolism. While other diagnostic tests may be helpful, a CT scan of the chest is more readily accessible to diagnose a PE.

The nurse is caring for a patient with complaints of progressive, ascending paralysis starting from the feet and progressing over the last week. It would be a PRIORITY for the nurse to monitor the patient's? A) blood glucose. B) peripheral pulses. C) pupillary response. D) respiratory status.

D The patient is exhibiting signs of Guillain-Barré Syndrome, of which the most severe complication is respiratory arrest. Pupillary response, peripheral pulses and blood glucose are part of the secondary assessment, but airway is the chief priority.

The nurse is caring for a patient with a long history of alcohol abuse who now has coffee-ground emesis and hypotension. The nurse should prepare for emergent administration of? A) thiamine. B) hypertonic saline. C) ondansetron (Zofran). D) blood products

D The patient is exhibiting signs of acute gastrointestinal bleeding (peptic ulcer disease, esophageal varices, etc.). The patient's active bleeding and hypotension indicates the need for the emergent administration of blood products. Thiamine and Zofran may be given, but are not the priority in this scenario. Hypertonic saline is not indicated.

A patient presents with audible wheezing and has an oxygen saturation of 88% on room air. The patient reports eating shrimp for lunch. It would be a PRIORITY for the nurse to administer? A) albuterol. B) oxygen. C) diphenhydramine. D) epinephrine.

D The patient is experiencing a life-threatening allergic reaction to shellfish. Epinephrine is the the priority medication to administer. The other medications may be given at a later time.

The nurse is assessing a patient who states that they are unable to sleep due to nightmares and flashbacks of an assault. The nurse knows that the BEST initial intervention is to? A) identify community resources. B) ask specific questions about the assault. C) quickly medicate the patient. D) provide a quiet environment.

D The patient is having post traumatic stress disorder. The patient should be assessed in a quiet environment. The patient may need medication but there is no indication to quickly medicate the patient, everyone should remain calm and relaxed. Do not ask specific questions about the initiating event, rather let the patient express their feelings and concerns. The patient must be assessed for the potential to harm themselves or others before community resources are provided.

A patient with pale, cool skin is noted to be difficult to arouse during assessment. Vital signs are: BP 89/53 mmHgHR 40 beats/minRR 22 breaths/min The patient has an ekg that shows a third degree heart block. The PRIORITY intervention is to? A) initiate synchronized cardioversion. B) administer atropine. C) administer amiodarone (Cordarone) D) initiate transcutaneous pacing.

D The patient is unstable based on the vital signs and patient assessment. Transcutaneous pacing will temporarily restore normal electrical and mechanical functions of the heart until the underlying cause of the heart block is treated or a permanent pacemaker is implanted. Atropine is not recommended to treat complete heart block. Synchronized cardioversion and amiodarone are used to treat tachycardic dysrhythmias.

A patient with pale, cool skin is noted to be difficult to arouse during assessment. Vital signs are: BP 89/53 mmHgHR 40 beats/minRR 22 breaths/min. The patient has an ECG that shows a third degree heart block. The PRIORITY intervention is to? A) initiate synchronized cardioversion. B) administer atropine. C) administer amiodarone (Cordarone) D) initiate transcutaneous pacing.

D The patient is unstable based on the vital signs and patient assessment. Transcutaneous pacing will temporarily restore normal electrical and mechanical functions of the heart until the underlying cause of the heart block is treated or a permanent pacemaker is implanted. Atropine is not recommended to treat complete heart block. Synchronized cardioversion and amiodarone are used to treat tachycardic dysrhythmias.

A patient presents with right-sided facial droop and slurred speech that started 2 hours ago. The patient's blood glucose level is 105 mg/dL and vital signs are as follows: BP 210/122 mmHgHR 72 beats/min RR 19 breaths/min O2 sat 98% on room air The nurse should FIRST anticipate an order for? A) administering tPA immediately. B) rechecking blood glucose. C) evaluation by a neurologist. D) antihypertensive medication.

D The patient's blood pressure needs to be under 185/110 mmHg prior to tPA being administered therefore administering an antihypertensive would occur first.

A patient who returns to the emergency department 3 days after a motor vehicle crash has pain in the left upper quadrant of the abdomen. Assessment reveals pain on palpation, and the patient is hemodynamically stable. The nurse should be MOST concerned about the possibility of? A) bleeding from a liver laceration. B) a pneumothorax. C) a costochondritis. D) a splenic hematoma.

D The spleen, located in the left upper abdominal quadrant, is the most commonly injured organ in patients with blunt abdominal trauma. Because the spleen (like the liver and kidneys) is an encapsulated organ, bleeding from the parenchyma is commonly well-contained within the capsule. Therefore, the patient remains hemodynamically stable and there is no free blood in the peritoneum. Most patients with a splenic subcapsular hematoma will heal nicely without intervention. However, the capsule can rupture, releasing blood into the abdominal cavity. A pneumothorax is manifested by respiratory findings, costochondritis is characterized by pain with inspiration or chest palpation, and a liver laceration causes pain in the RIGHT upper abdominal quadrant.

Following a snake bite, a patient arrives to the emergency department in severe pain with two puncture wounds to the upper extremity. After observing swelling at the site, the nurse's NEXT step is to? A) suction the puncture wound sites. B) apply a tourniquet to the upper extremity. C) apply an ice pack to the bite site. D) immobilize the extremity below the level of the heart.

D Therapeutic interventions for snakebites include immobilizing the extremity at or below the level of the heart, removing constricting clothing or jewelry, pain management, and administration of antivenom as indicated. Application of ice, suctioning of the wound and placement of a tourniquet are all contraindicated in the treatment of snake bites and may cause worsening outcomes if used in this patient's care.

An 11-year-old patient is admitted to the emergency department one hour after receiving a blow to the right eye. The patient is unable to look upward with the injured eye. The nurse should suspect a? A) hyphema. B) traumatic infarction of the optic nerve. C) vitreous hemorrhage. D) blowout fracture.

D These symptoms are caused by the ocular motor nerve being trapped within the fractured area of the orbit.

A school-age patient presents after a house fire with 3rd degree circumferential burns of the neck and chest. Physical exam reveals the extent of the burn is 60% total body surface area, stridor, drooling and somnolence. The patient's vitals signs are as follows: Blood Pressure 90/45 mmHgHeart Rate 133 beats/minRespirations 28 breaths/minSpO2 100% on room airOral Temperature 99.5FGlasgow Coma Scale (GCS) of 14 The ED nurse understands the PRIORITY intervention is to? A) Apply warm saline dressings B) Administer IV fluids C) Perform wound debridement D) Prepare for intubation

D This patient is demonstrating signs and symptoms of smoke inhalation and younger patients are at greater risk for compromised airway. Due to risk for airway obstruction, the priority intervention is to protect the airway and initiate early intubation. IV fluids, warm dressings and wound debridement are indicated for burn resuscitation, but these are not the priority interventions.

A patient presents with a partial amputation of the left lower extremity from a boat propeller. They arrive one hour post incident with a tourniquet in place, and massive transfusion protocol has been initiated. Current vital signs include a heart rate of 145 beats/min and blood pressure of 84/62 mmHg. The NEXT priority action is to? A) Target blood pressure goal of 120 mmHg systolic B) Bolus an isotonic crystalloid solution C) Initiate intravenous antibiotics D) Administer tranexamic acid (TXA)

D Tranexamic acid (TXA) should be used when encountering the patient with massive hemorrhage and/or receiving blood transfusion. TXA is a medication which prevents the breakdown of fibrin and the breakdown of clots that have formed. Studies have shown that, if given within 3 hours of injury, administration of intravenous TXA has significantly reduced mortality. Rapid bolus of an isotonic crystalloid solution can cause hemodilution and break clots that have already formed. Blood pressure drives bleeding; research supports permissive hypotension of a systolic blood pressure targeted to no higher than 90 to 100 mm Hg. Antibiotics should be considered in treatment but are not priority.

The FIRST step in the triage assessment of a patient with a behavioral health concern is the: chief complaint. environment of care checklist. health history questionnaire. across the room assessment.

D Triage of the patient with a behavioral health concern begins with the across-the-room assessment, which can provide the triage nurse important information. Health history, the environment of care checklist, and the chief complaint are important but not as important as the initial across the room survey. Bizarre dress, loud or rapid speech, odor of alcohol, demeanor, and activity level can often be determined by an across the room survey.

Which of the following is the MOST likely early complication of myocardial infarction? A) elevated blood pressure B) ruptured myocardium C) congestive heart failure D) ventricular dysrhythmia

D Ventricular dysrhythmias are due to the decrease in myocardial perfusion and oxygenation. This will cause an increase in irritability of the heart muscle. This is a common complication that usually occurs within 72 hours of myocardial injury.

What is the treatment of choice for testicular torsion? A) antibiotics B) surgical intervention C) bed rest D) scrotal sling

Immediate surgical exploration, within 4 to 6 hours of onset, with subsequent orchidopexy is routinely the treatment of choice for testicular torsion. If left untreated for 24 hours, the testicle has sustained necrosis.

A patient suspected of an overdose is unresponsive, hypotensive, bradycardic, and appears post ictal. Blood glucose is within normal limits. The nurse suspects? A) Febrile Seizure B) ETOH withdrawal C) Propanolol overdose D) Lisinopril overdose

Propanolol is a beta blocker with the highest rate of mortality due to it's lipophilic nature, allowing it to permeate the CNS. Seizures are common only with Propanolol. ETOH withdrawal would cause tachycardia, Ace Inhibitors do not typically affect heart rate, and febrile seizures are most common in children under five years old.

The emergency nurse is assessing a patient with fluid volume overload that has a history of renal failure. The patient has missed dialysis treatment and is complaining of a headache, dizziness and confusion. The nurse is concerned for? A) hypernatremia. B) hypermagnesemia. C) hypomagnesemia. D) hyponatremia.

The patients symptoms are caused by hyponatremia which is a common finding associated with fluid volume overload and renal failure. Hypernatremia is caused by volume depletion, not fluid volume overload. Hypomagnesemia is associated with excessive kidney excretion of magnesium or over use of diuretics. Hypermagnesemia is associated with renal failure patients however seen in patients when the magnesium rich diasylate from treatment causes excessive intake resulting in muscle weakness, decreased deep tendon reflexes, decreased level of consciousness, and respiratory muscle paralysis.


Conjuntos de estudio relacionados

Data Resource Management Exam 1 Chapter 5

View Set

Human Capital: INVESTMENTS (Before Midterm)

View Set

Life Insurance Quiz Practice Questions

View Set

Ch58: Disorders of the Kidneys and Ureters

View Set

Chapt 14 Ancient Mediterrranean Worlds

View Set